Solution Report - 1 File Download

86
Copyright © 2014 Delhi Academy of Medical Sciences, All Rights Reserved. 1/86

Transcript of Solution Report - 1 File Download

Copyright © 2014 Delhi Academy of Medical Sciences, All Rights Reserved. 1/86

Copyright © 2014 Delhi Academy of Medical Sciences, All Rights Reserved. 2/86

Test Information Test Name SWTS-SURGERY-1 2017(MDMS) Total Questions 200

Test Type Examination Difficulty Level Difficult

Total Marks 600 Duration 120minutes

Test Question Language:- ENGLISH

(1). Which of the following statements regarding glioblastoma multiforme is true?

a. It is a neuronal cell tumor

b. It arises from the malignant degeneration of an astrocytoma

c. With aggressive treatment, most patients can live up to 10 years with this disease

d. It is the most common childhood intracranial neoplasm

Solution. (b) It arises from the malignant degeneration of an astrocytoma References: Read the text below. Sol:- Glioblastoma multiforme is the most common form of primary intracranial neuroepithelial tumor.- It represents 25% of all intracranial tumors and 50% of tumors originating in the central nervous system. It is a heterogeneous glial celltumor derived from the malignant degeneration of an astrocytoma or anaplastic astrocytoma.- These tumors are most commonly found in the cerebral hemispheres during the fifth decade of life.- CT and MRI scans typically reveal an irregular lesion with hypodense central necrosis, peripheral ring enhancement of the highlycellular tumor tissue, and surrounding edema and mass effect.- Curative resections are rare. Therapy consists of diagnostic biopsy followed by radiotherapy to slow the tumor growth.- The course of the disease progresses rapidly after presentation, with few patients living more than 2 years.

Correct Answer. b

(2). The most common cause of acquired arterio venous fistula is :

a. Bacterial infection

b. Fungal infection

c. Blunt trauma

d. Penetrating trauma

Solution. (d) Penetrating traumaRef.: Read the text belowSol :- Arteriovenous fistula- Abnormal communication between artery and vein either congenital or acquired- Most common cause of AV fistula is penetrating trauma- Through AV fistula high pressure arterial blood flows to vein.

Correct Answer. d

Copyright © 2014 Delhi Academy of Medical Sciences, All Rights Reserved. 3/86

(3). False statement about Thyroglossal cyst is?

a. Remnant of 2nd brancheal arch

b. May contain total thyroid tissue in 10% cases

c. Papillary cancer be found in the cyst

d. Attached to base of tongue with a tract that goes through body of hyoid

Solution. (a) Remnant of 2nd brancheal archRef: Read the text belowSol:

Correct Answer. a

(4). Pain along medial aspect of arm in a post mastectomy patient is due to?

a. Phantom breast pain

b. Intercostobrachial neuralgia

c. Neuroma pain

d. Other nerve injury pain

Solution. (b) Intercostobrachial neuralgiaRef.: Read the text belowSol :- Intercostobrachial neuralgia is a rare but very bothersome nerve pain issue.- This is a nerve pain issue in which nerves coming from the upper thoracic spine and going into your armpit upper inner arm, and upperchest wall are damaged and cause pain in this region.- Intercostobrachial neuralgia is most common in people (mostly women) who have had breast surgery for cancer such as mastectomy.This is known as post mastectomy pain.- It also happens after radiation therapy to the lungs and upper chest wall. Trauma to the ribs such as rib fractures can causeintercostobrachial neuralgia.

Correct Answer. b

Copyright © 2014 Delhi Academy of Medical Sciences, All Rights Reserved. 4/86

(5). Bochdalek hernia is seen through?

a. Mediastinum

b. Urogenital diaphragm

c. Diaphragm

d. Dura

Solution. (c) DiaphragmRef.: Read the text belowSol :- In infants with Bochdalek type congenital diaphragmatic hernia the severity of pulmonary hypoplasia and the resultant pulmonaryhypertension are key determinants of survival.- Barotraumas and hypoxia should be avoided.

Correct Answer. c

(6). Which of the following statements regarding the Glasgow coma scale is true?

a. It serves as a scale to assess the long-term sequelae of head trauma

b. It includes measurement of intracranial pressure

c. It includes measurement of papillary reflexes

d. It includes measurement of verbal response

Solution. (d) It includes measurement of verbal responseReferences: Read the text below.Sol:- The Glasgow coma scale was developed to enable an initial assessment of the severity of head trauma.- It is now also used to standardize serial neurologic examinations in the early postinjury period.It measures the level of consciousnessusing three parameters: verbal response (5 points), motor response (6 points), and eye opening (4 points). The score is the sum of thehighest number achieved in each category.- The fully oriented and alert patient will receive a maximum score of 15. A score of less than 5 is associated with a mortality of over 50%.

Correct Answer. d

(7). Treatment of high grade mucoepidermoid tumour of salivary gland is?

a. Wide local excision.

b. Wide local excision + Regional node dissection.

c. Wide local excision + Regional node dissection + Radiotherapy.

d. Wide local excision + Regional node dissection + Radiotherapy + Chemotherapy.

Solution. (c) Wide local excision + Regional node dissection + Radiotherapy.Ref: Sabistom 18/e, p 1273Sol :- For Low grade malignant tumours excision of the tumor with complete parotid (with clear margins) is the aim.- No adjuvant chemotherapy is required.- For high grade cancers after generous primary excision, for clinically palpable nodes radical or modified radical neck dissection isperformed followed by radiotherapy in selected group.

Correct Answer. c

Copyright © 2014 Delhi Academy of Medical Sciences, All Rights Reserved. 5/86

(8). Most common malignant tumour of minor salivary gland is?

a. Mucoepidermoid carcinoma.

b. Adenocystic carcinoma.

c. Adenocarcinoma.

d. Malignant mixed tumour.

Solution. (b) Adenocystic carcinoma.Ref: Sabistom - 1273Sol :Adenoid cystic carcinoma: It is rare in parotid but commonest in minor salivary gland tumour, thus one fourth of malignant salivary glandtumours. 70% of all minor salivary gland tumor occur in oral cavity, mainly in hard palate. Adenoid cystic carcinoma has propensity forperineural invasion

Correct Answer. b

(9). Injury to external Laryngeal nerve will lead to?

a. Hoarseness

b. Loss of Timbre

c. Vocal cord lying in adducted position

d. None

Solution. (b) Loss of TimbreRef: Read the text belowSol:- The thyroid gland has an abundant blood supply with normal flow rate of 5 ml/g/ min. Although the thyroid represents about 0.4% ofbody weight it accounts for 2% of total blood flow.- The superior thyroid arteries arise from the external carotid and reach the upper poles of the thyroid and inferior pair spring from thethyrocervical trunk of the subclavian arteries and enter the lower poles from behind.- Frequently, a fifth artery, the thyreoidea ima, from the arch of the aorta, enters the thyroid in the midline (3%).- The 3 pairs of veins emerge from the interior of the gland and form a plexus of vessels under the capsule. These drain into the internaljugular, the brachiocephalic, and occasionally the anterior jugular veins.- RL nerve in injured at lower pole and external laryngeal nerve is injured at upper pole.- External Laryngeal nerve injury will lead to paralysis of cricothyroid leading to loss of tensor action of vocal cord thus voice will be oflow pitch.

Correct Answer. b

(10). In diabetic foot, which amputation is contraindicated?

a. Ray amputation

b. Forefoot amputation

c. Syme’s amputation

d. Below knee amputation

Solution. (c) Syme’s amputationRef.: Bailey & Love’s short practice of Surgery - 915Sol :- For any amputation to heal the local residual stump needs good vascularity, in syme’s amputation it is essential to preserve the bloodsupply to the heel flap by meticulous clean dissection of the calcaneus.- The tibia and fibula are sectioned as low as possible to the top of the joint.- This type of procedure is rarely suitable for severely ischemic athermanous limbs because of the poor healing of the heel flap.- Hence not suitable for diabetic foot so it is contraindicated.

Correct Answer. c

Copyright © 2014 Delhi Academy of Medical Sciences, All Rights Reserved. 6/86

(11). Second Commonest parotid tumor is?

a. Pleomorphic tumor

b. Warthin’s tumor

c. Adenoid cystic Ca

d. Secondaries

Solution. (b) Warthin’s tumorRef: Read the text belowSol :- Most salivary gland tumors are benign and by far the most common site is the parotid gland. 80% of parotid tumors are benign, ofwhich 80% are pleomorphic adenomas, and 80% arise from the superficial lobe of the parotid.- There is a greater chance of malignancy in the submandibular and sublingual glands with 50% being benign. Other benign tumorsincluded in the differential include Warthin's Tumor, acidophilic cell adenoma, and monomorphic adenoma. Adequate resection of benignsalivary gland lesions provides a 97-98% control rate.- Warthin's tumor is the second most common benign lesion in the parotid gland. These represent 2-10% of all parotid gland tumors.

Correct Answer. b

(12). False about lobular carcinoma breast is?

a. Presents as breast mass

b. Frequently bilateral

c. Poor Prognosis

d. Multicentre

Solution. (c) Poor PrognosisRef: Read the text belowSol :- “The presenting symptom in most cases of lobular carcinoma is breast mass with ill defined margins, but in some cases the onlyevidence of neoplasm is vague thickening or fine diffuse nodularity.- Large lesions are more likely to cause skin retraction and fixation.- It is not prone to calcification or paget’s disease.

Correct Answer. c

(13). Most common cause of entero-cutaneous fistula is?

a. Spontaneous

b. Crohn’s disease

c. Tuberculosis

d. Post-operative

Solution. (d) Post-operativeRef: Read the text belowSol :Enterocutaneous or faecal fistula- Enterocutaneous fistulae rarely occur spontaneously and usually follow previous surgery.- Crohn’s disease of small bowel is a common cause of enterocutaneous fistulas.- The most common cause of enterocutaneous fistula is, however, previous surgery.- Radiation damage is also another cause of fistula formation.

Correct Answer. d

Copyright © 2014 Delhi Academy of Medical Sciences, All Rights Reserved. 7/86

(14). A 2 cm sized benign thyroid nodule causing pressure symptom in a 35 year old female is best treated by?

a. Near Total thyroidectomy

b. Hemithyrodectomy

c. Subtotal thyroidectomy

d. Lobectomy

Solution. (b) HemithyrodectomyRef: Read the text belowSol:- A Solitary Thyroid Nodule commonly occurs (4% to 7% of adults) and is usually benign. Such nodules may be associated with a goiter orwith an otherwise normal thyroid.- Considering the malignant potential of the newly discovered thyroid nodule surgical intervention is appropriate for all malignant orsuspicious thyroid nodules and for all large (>3 or 4 cm) nodules. The frequency of cancer in surgically excised nodules is 8% to 17%.- Best surgery for Benign thyroid nodule is Hemithyroidectomy

Correct Answer. b

(15). Dercrum’s disease is characterized by?

a. Lipodermatosclerosis

b. Tender subcutaneous lipomas

c. Morbid obesity

d. None

Solution. (b) Tender subcutaneous lipomasRef: Read the text belowSol :- Adiposis dolorosa (Dercum’s disease) (acute panniculitis) was first described in 1892 and is characterized by progressive, multiple,painful, circumscribed subcutaneous lipoma, generalized obesity in women of menopausal age, weakness and frequent tendency tofatigue and mental phenomenon (eg. Emotional instability, depression, epilepsy, dementia).- It may have a familial basis.

Correct Answer. b

(16). Recurrent peri areolar abscess is seen in?

a. Duct papilloma

b. Paget’s disease

c. Periductal mastitis

d. Fibroadenoma

Solution. (c) Periductal mastitisRef.: Read the text belowSol :- Periductal mastitis is an inflammatory condition of the subareolar ducts; the cause is unknown. Periductal mastitis primarily affectsyoung women but can occur in men as well.- The majority of patients with periductal mastitis are smokers. It has been postulated that smoking is associated with damage of thesubareolar ducts, with tissue necrosis and subsequent infection- Periductal mastitis is also associated with squamous metaplasia, which is likely a consequence of ongoing inflammation. It has beensuggested that squamous metaplasia may lead to partial duct obstruction with subsequent dilatation and secondary inflammation andinfection

Correct Answer. c

Copyright © 2014 Delhi Academy of Medical Sciences, All Rights Reserved. 8/86

(17). What is true of cystic hygroma?

a. It arises from sweat glands in the neck.

b. It is usually an anterior midline structure.

c. It may occur in the mediastinum.

d. Its lesions are usually easy to enucleate.

Solution. (c) It may occur in the mediastinum.Ref:Read the text belowSol:- Cystic hygromas are relatively rare tumors.- Most are encountered in the posterior triangle of the neck, but occasionally they are found in the mediastinum, axilla, or groin.- They are often noted at birth and represent persistence of primary lymphatic buds.- They extend into the surrounding tissues but are not associated with malignancy.- Transillumination is a useful sign to diagnose this lesion.

Correct Answer. c

(18). A 25-year-old man fell down from his bicycle and hit a concrete wall on his left side. An ultrasound examination showed free fluid in theabdomen. A CT scan confirmed a grade III splenic injury. The most important contraindication for a nonoperative management of thesplenic injury is:

a. Hemodynamic instability

b. Active bleeding on CT scan

c. Adult patient

d. Lack of availability of blood for transfusion

Solution. (a) Hemodynamic instabilityRef: Read the text belowSol :- While all other choices may be relative, hemodynamic instability is the prime contraindication for nonoperative treatment.

Correct Answer. a

(19). A 60-year-old man is attacked with a baseball bat and sustains multiple blows to the abdomen. He presents to the emergency departmentin shock and is brought to the operating room (OR), where a laparotomy reveals massive hemoperitoneum and a stellate fracture of theright and left lobes of the liver. Which of the following techniques should be used immediately?

a. Pringle’s maneuver

b. Packing the liver

c. Suture ligation

d. Ligation of the right hepatic artery

Solution. (b) Packing the liverRef: Read the text belowSol :- The initial operative step is packing of the liver to obtain control of the bleeding.- A Pringle maneuver can then be performed.- In this procedure, the proper hepatic artery is compressed between one finger inserted into the foramen omentalis (Winslow, epiploic)and another anterior to the free edge of the lesser (gastrohepatic omentum.- Selective right hepatic artery is rarely useful, and ligation of the proper hepatic artery is contraindicated.

Correct Answer. b

Copyright © 2014 Delhi Academy of Medical Sciences, All Rights Reserved. 9/86

(20). Boundaries of Behr’s triangle is formed by all except?

a. Inferior thyroid artery

b. Common carotid artery

c. Cricoythyroid muscle

d. Recurrent laryngeal nerve

Solution. (b) Cricoythyroid muscleRef: Read the text belowSol:

Correct Answer. b

(21). A 43-year-old male develops cancer of the lip.What is true of this condition?

a. It involves the upper lip in 90% of patients.

b. It is more common at the lateral commissure than in the middle.

c. It usually occurs beyond the vermilion border.

d. It results in cure in about 60% of cases.

Solution. (c) It usually occurs beyond the vermilion border.Ref:Read the text belowSol:- If the lesion is treated early, patients will achieve a cure in most cases. The upper lip is involved in 10% of patients.

Correct Answer. c

Copyright © 2014 Delhi Academy of Medical Sciences, All Rights Reserved. 10/86

(22). Which is the best fluid for resuscitation during shock state?

a. Crystalloids

b. Colloids

c. Plasma substitutes

d. 5% dextrose

Solution. (a) CrystalloidsRef.: Read the text belowSol :- “Fluid resuscitation is a major adjunct to physically controlling hemorrhage in patients with shock.- The ideal type of fluid to be used continues to be debated; however, crystalloids continue to be the mainstay of fluid choice”- Several studies have demonstrated increased risk of death in bleeding trauma patients treated with colloid compared to patientstreated with crystalloid. In patients with severe hemorrhage, restoration of intravascular volume should be achieved with blood products.

Correct Answer. a

(23). Drug of choice for estrogen receptor positive breast cancer is?

a. Transtuzumab

b. Taxmoxifen

c. Cyproterone acetate

d. Anastrozole

Solution. (b) TaxmoxifenRef.: Read the text belowSol :- “The drug tamoxifen is an estrogen agonist – antagonist and currently the first – line treatment of estrogen – sensitive breast cancer”- Tamoxifen can replace oophorectomy in premenopausal women with ER-positive metastatic cancer, and it is considered the drug of firstchoice in both premenopausal and postmenopausal patients with ER-positive breast cancer.

Correct Answer. b

(24). Patient presents with loss of consciousness CT show multiple spotty hemorrhages and full basal cisterns:

a. Brain contusions

b. Diffuse axonal injury

c. Subdural hematoma

d. Multiple infarcts

Solution. (b) Diffuse axonal injuryRef.: Grainger Radiology - 2398Sol :Diffuse axonal injury- Brain injury resulting from mechanical shearing at the gray-white matter interface secondary to severe acceleration deceleration typeforces.- This causes disruption and tearing of, myelin sheath, axon and blood capillaries.- CT findings include diffuse cerebral swelling, small hemorrhages can be seen in more severe cases hemorrhage is classically seen in thecorpus collasum and the dorsolateral midbrain.- Lesions are usually multiple range from 0.5 to 1.5 cm and are typically oval or elliptical.

Correct Answer. b

Copyright © 2014 Delhi Academy of Medical Sciences, All Rights Reserved. 11/86

(25). A 24-year-old man falls on the ground when is struck in the right temple by a baseball. While being driven to the hospital, he lapses intocoma. He is unresponsive with the dilated right pupil when he reaches the emergency department. The most important step in initialmanagement is :

a. Craniotomy

b. CT scan of the head

c. X-ray of the skull and cervical spine

d. Doppler ultrasound examination of the neck.

Solution. (b) CT scan of the headRef.: Schwartz 8/e, 1618Sol :For any patient with head injury an emergency CT scan is indicated before surgical intervention. Guidelines for CT in Head injury:- GCS < 13 at any point of time- GCS 13 or 14 at 2 hours- Presence of focal neurologic deficit- Suspected skull fracture- Seizures- More than one episode to vomiting- Urgent CT scan is needed if none of the above but:- Age > 65- Coagulopathy (e.g. patient is taking anticoagulants)- Antegrade amnesia > 30 min (CT with in 8 hours)- Dangerous mechanism of injury.

Correct Answer. b

(26). Treatment of choice for a patient presenting with venous ulcer and incompetent perforators is?

a. Stripping of saphenous vein

b. Subfacial ligation of perforators

c. Saphenofemoral ligation

d. Conservative

Solution. (b) Subfacial ligation of perforatorsRef.: Read the text belowSol :- Saphenofemoral flush ligation with stripping of saphenous vein is used for saphenofemoral incompetence while Subfacial ligation ofperforators is sued for perforator incompetence.- Hence in this case subfacial ligation of the perforators will be the treatment of choice.- A significant proportion of patients with venous ulceration have normal function in the deep veins, and surgical treatment is a usefuloption that can definitively address the hemodynamic derangements.

Correct Answer. b

Copyright © 2014 Delhi Academy of Medical Sciences, All Rights Reserved. 12/86

(27). A patient came with venous ulcer, on examination there is edema and tenderness. The following are used except:

a. Subfascial ligation of perforators

b. Stripping of large saphenous vein

c. Skin substitutes

d. Radiotherapy

Solution. (d) RadiotherapyRef.: Read the text belowSol :The main aim of the treatment is to create such an environment that allows skin to grow across an ulcer. In the majority of cases thisrequires finding and treating underlying venous reflux and NICE (National Institute for Health and Care Excellence) recommendsreferral to a vascular service for anyone with a leg ulcer that has not healed within 2 weeks or anyone with a healed leg ulcer1. Compression therapy Non-elastic, ambulatory, below knee (BK) compression counters the impact of reflux on venous pump failure.Compression therapy is used for venous leg ulcers and can decrease blood vessel diameter and pressure, which increases theireffectiveness, preventing blood from flowing backwards2. Pentoxifylline Pentoxifylline is a useful add on treatment to compression stockings and may also help by itself. It works by reducingplatelet aggregation and thrombus formation.3. Skin grafts and artificial skin Two layers of skin created from animal sources as a skin graft has been found to be useful in venous legulcers4. Subfascial ligation of perforators5. Stripping of large saphenous vein

Correct Answer. c

(28). Keloid formation is not seen over?

a. Ear

b. Face

c. Eyelid

d. Neck

Solution. (c) EyelidRef.: Read the text belowSol :- Keloids are defined as scars that grow beyond the borders of the original wounds, and these scars rarely regress with time. Keloids aremore prevalent in patients with more darkly pigmented skin; they develop in 15% to 20% of African Americans, Asians, and Hispanics.- Keloids appear to have a genetic predisposition. Keloid scars tend to occur above the clavicles, on the trunk, on the upper extremities,and on the face. They cannot be prevented at this time and are often refractory to medical and surgical intervention.- Hypertrophic scars, in contrast, are raised scars that remain within the confines of the original wound and frequently regressspontaneously.- A hypertrophic scar can occur anywhere on the body.- Keloids most frequently develop on the upper back and chest, neck shoulders, jaw, ear lobes and legs. These areas all share the qualityof high tension on healing wounds.- Keloids rarely appear on the eyelids, genitals, palms and soles.

Correct Answer. a

Copyright © 2014 Delhi Academy of Medical Sciences, All Rights Reserved. 13/86

(29).

This is an example of

a. Reef Knot

b. Granny Knot

c. Surgical Knot

d. Extra half hitch knot

Solution. (a) Reef KnotRef: Read the text belowSol:

Correct Answer. a

Copyright © 2014 Delhi Academy of Medical Sciences, All Rights Reserved. 14/86

(30). Massive splenomegaly is seen in?

a. Hairy cell leukaemia

b. Acute leukaemia

c. Enteric fever

d. Multiple myeloma

Solution. (a) Hairy cell leukaemiaRef.: Read the text belowSol :- Splenomegaly is a common finding in a wide spectrum of diseases. Massive splenomegaly, however, always indicates underlying pathology.- Massive splenomegaly is usually defined as a spleen extending well into the left lower quadrant or pelvis or which has crossed the midline of theabdomen.- Massive spleens weigh at least 500 to 1000 g.- The most common disorders associated with splenomegaly were hematologic, infectious, hepatic, congestive, and inflammatory.

Correct Answer. a

(31). Hunterian ligature is for?

a. A-V fistula

b. Aneurysm

c. Lymphangitis

d. Varicose veins

Solution. (b) AneurysmRef.: Read the text belowSol :- Arterial ligation was popularized in the 1800s by John Hunter, who demonstrated safe and reproducible means of ligating certainperipheral arteries.- Hunterian ligation, named in his honor, was adopted by many surgeons for the treatment of aneurysms of the ICA

Correct Answer. b

Copyright © 2014 Delhi Academy of Medical Sciences, All Rights Reserved. 15/86

(32). In the breast conservative surgery which of the following investigation is required?

a. Serum calcium

b. Total body scan

c. Sentinel node biopsy

d. Tumor markers

Solution. (c) Sentinel node biopsyRef.: Read the text belowSol :- Breast conservation surgery is currently the standard treatment for women with stage 0, I, or II invasive breast cancer.- Breast conservation therapy (BCT) is partial mastectomy.Lumpectomy and SLNB (or axillary lymph node dissection followed by breastirradiation.- Sentinel lymph node dissection is now the preferred staging procedure with a clinically node-negative axilla.- When the procedures are sequenced in the operating room, the sentinel node procedure usually is performed before removal of theprimary breast tumor.- When indicated, intraoperative assessment of the sentinel node can proceed while the segmental mastectomy is being performed. Whenthe sentinel lymph node does not contain metastatic disease, axillary lymph node dissection is avoided. It is the surgeon’s responsibilityto ensure complete removal of cancer in the breast.- Ensuring surgical margins that are free of breast cancer will minimize the chances of local recurrence and will enhance cure rates.

Correct Answer. c

(33). Cause of death in golden hours (within 6 hours) in acute trauma :

a. Airway obstruction

b. Tension pneumothorax

c. Pericardial tamponade

d. Pericardial effusion

Solution. (a) Airway obstructionRef.: Sabiston - 495Sol :

Correct Answer. a

Copyright © 2014 Delhi Academy of Medical Sciences, All Rights Reserved. 16/86

(34). Bowel part commonly injured in blunt abdominal injury is/are:

a. Mid ileum

b. Proximal jejunum

c. Mid jejunum

d. Distal ileum

Solution. (b) Proximal jejunumRef.: Schwartz - 173Sol :Small Bowel injury- Small bowel is the most frequently injured organ after penetrating abdominal injuries.- Injury of bowel during blunt trauma is likely to involve either the fixed part of gut (iliocaecal junction) or where the fixed part joinsmobile part (duodenojejunal flexure)- CT can not reliably diagnose SI injury- Most injury are treated with a lateral single layer running suture.- In case of multiple injury resection with end to end anastomose is the preferred treatment.

Correct Answer. b

(35). A70-year-old man with a long-standing history of diabetes develops gangrene of the right second toe. What is true of his diabetic foot?

a. Dorsalis pedis and posterior tibial arteries are always absent.

b. Gangrene of the toe always requires urgent below-knee amputation.

c. Arterial reconstruction is invariably required.

d. Trophic ulcers are sharply demarcated.

Solution. (d) Trophic ulcers are sharply demarcated.Ref: Read the text belowSol :- Patients with a diabetic foot may have localized arterial occlusion involving the popliteal artery and its branches, usually sparing thefemoral artery.- Although patients have gangrene of the toes, there may be a palpable pulse in the foot. In the presence of localized disease, trophiculcers and even gangrene of the toes may respond to local foot care, and major vascular reconstruction or amputations are not required.- The trophic ulcers have punched sides.- Patients may not realize the gravity of localized gangrene with spreading cellulitis, which develops because of the neurotropic nature ofthe lesions with the absence of pain sensation

Correct Answer. d

(36). Eleven years after undergoing right modified radical mastectomy, a 61-year-old woman develops raised red and purple nodules over theright arm. What is the most likely diagnosis?

a. Lymphangitis

b. Lymphedema

c. Lymphangiosarcoma

d. Hyperkeratosis

Solution. (c) LymphangiosarcomaRef: Read the text belowSol :- Lymphangiosarcoma is a rare complication of long-standing lymphedema, most frequently described in a patient who has previouslyundergone radical mastectomy (Stewart-Treves syndrome).- It usually presents as blue, red, or purple nodules with satellite lesions. Early metastasis,mainly to the lung, may develop if it is notrecognized early and widely excised.- Lymphedema is a complication of radical mastectomy and presents as diffuse swelling and nonpitting edema of the limb.- Lymphangitis and hyperkeratosis are complications of lymphedema.

Correct Answer. c

Copyright © 2014 Delhi Academy of Medical Sciences, All Rights Reserved. 17/86

(37). Four days after undergoing subtotal gastrectomy for stomach cancer, a 58-year-old woman complains of right leg and thigh pain,swelling and redness, and has tenderness on examination. The diagnosis of deep vein thrombosis is entertained. What is the initial test toestablish the diagnosis?

a. Venography

b. Venous duplex ultrasound

c. Impedance plethysmography

d. Radio-labeled fibrinogen

Solution. (b) Venous duplex ultrasoundRef: Read the text belowSol :- The most accurate method of confirming the diagnosis of venous thrombosis is the injection of contrast material to visualize the venoussystem (venography).- However, this method is invasive and time-consuming and must be done in the radiology suite.- Venous duplex ultrasound is noninvasive, can be done bedside, and has a sensitivity and specificity of 96 and 100%, respectively.- The other methods listed are used less often in certain selected patients.

Correct Answer. b

(38). A 25 year old female complaints of discharge of blood from single duct of breast. The most appropriate treatment is:

a. Radical duct excision

b. Microdochectomy

c. Radical mastectomy

d. Biopsy to rule out carcinoma

Solution. (b) MicrodochectomyRef.: Sabiston - 867Sol :Duct papilloma- Benign polyp of breast ducts, usually single and unilateral, particularly seen in middle aged women- Presents with small swelling just beneath the areola, palpation of which results in discharge of blood from nipple.- Tumor is situated in one of the large lactiferous duct.- Large duct papilloma is not associated with increased risk of cancer while small duct papilloma (usually multiple and deeper) increasethe risk (relative risk 1.52-2.0)Diagnosed by cytology. Treatment- Microdochectomy – treatment of choice- Cone excision of major duct (Had field operation) when duct of origin of nipple bleeding is uncertain or when there is bleeding frommultiple ducts.

Correct Answer. b

Copyright © 2014 Delhi Academy of Medical Sciences, All Rights Reserved. 18/86

(39). Green discharge is most commonly seen with:

a. Duct papilloma

b. Duct ectasia

c. Retention cyst

d. Fibroadenosis

Solution. (b) Duct ectasiaRef.: Bailey 25/e 831Sol :

Correct Answer. b

(40). The tumor, which may occur in the residual breast or overlying skin following wide local excision and radiotherapy for mammarycarcinoma is :

a. Leiomyosarcoma

b. Squamous cell carcinoma

c. Basal cell carcinoma

d. Angiosarcoma

Solution. (d) AngiosarcomaRef.:Sabiston 18/eSol :- “Angiosarcoma and lymphangiosarcoma may develop as a complication of chronic lymphedema specially in patients of breast cancer”.Angiosarcoma- Vascular tumor arising in dermis after breast irradiation or in the lymphedematous upper extremity.- Rarely it occur denovo too.- Present as postirradiaton vascular proliferation. The differential diagnosis is atypical vascular proliferation.- Histologically tumor composed of anastomosing tangle of blood vesels in the dermis and subcutaneous fat.- Clinically present as reddish brown to purple raised rash within the radiation portals and on the skin of breast.- Treatment includes surgery to ensure negative margins followed by split skin graft or myocutaneous flap.- Lymphatic metastasis is very rare and lymphatic resection is not required.

Correct Answer. d

Copyright © 2014 Delhi Academy of Medical Sciences, All Rights Reserved. 19/86

(41). After trauma, hypovolemic shock can be due to all except:

a. Pelvic fracture

b. Head injury

c. Blunt trauma to abdominal viscera

d. Hemothorax

Solution. (b) Head injuryRef.: Read the text belowSol :- Losing about 1/5 or more of the normal amount of blood in your body causes hypovolemic shock- Intracranial injuries alone do not produce circulatory inadequacy until brain stem and its reticular activating system is profoundlyinvolved.- Therefore the presence of shock in a head injury patient indicates a search for another cause.

Correct Answer. b

(42). This is an example of which type of De Bakey Artic dissection?

a. Type I

b. Type II

c. Type III

d. Type B

Solution. (c) Type IIIRef: Read the text belowSol:

Correct Answer. c

Copyright © 2014 Delhi Academy of Medical Sciences, All Rights Reserved. 20/86

(43). Non Absorbable suture among the following is?

a. Polydiaxanone

b. Polyester

c. Polyglactin

d. Monocryl

Solution. (b) PolyesterRef.: Read the text belowSol :

Correct Answer. b

(44).

An adult patient with leg pain and gangrene of toe. His ankle to brachialarterial pressure ratio would be less than?

a. 1

b. 0.3

c. 0.5

d. 0.8

Solution. (b) 0.3Ref.: Read the text belowSol :

Correct Answer. b

Copyright © 2014 Delhi Academy of Medical Sciences, All Rights Reserved. 21/86

(45). Ca breast in females is least related to:

a. BRCA-1

b. BRCA-2

c. Li fraumeni syndrome

d. Ataxia telangiectasia

Solution. (d) Ataxia telangiectasiaRef.:Robbins - 1134Sol :

Correct Answer. d

(46). Post splenectomy infections are common in

a. 0 – 4 yrs

b. 5 – 12 yrs

c. Above 30 yrs

d. 13 – 30 yrs

Solution. (a) 0 – 4 yrsRef: Acute Care Surgery - 513Sol :- Post splenectomy infections are common in 0 – 4 yrs

Correct Answer. a

(47). Which of the following is goitrogenic

a. Cauliflower

b. Potato

c. Carrot

d. Brinjal

Solution. (a) CauliflowerRef: Harrison’s- 335Sol :- Endemic goiter is also caused by exposure to environmental goitrogens such as cassava root, which contains a thiocyanate; vegetablesof the Cruciferae family e.g., Brussels sprouts, cabbage, and cauliflower; and milk from regions where goitrogens are present in grass.

Correct Answer. a

Copyright © 2014 Delhi Academy of Medical Sciences, All Rights Reserved. 22/86

(48). On the 7th post-operative day, abdominal wound shows pink sero sanguinous discharge. It suggests?

a. Impending wound dehiscence

b. Infection in the abdomen

c. Stitch abscess

d. Healing wound

Solution. (a) Impending wound dehiscenceRef.: Read the text belowSol :- By Sharon Baranoski, Elizabeth A. Ayello Ph. D. 2nd ed page 379- Acute wound failure (wound dehiscence or a burst abdomen) refers to postoperative separation of the abdominal musculoaponeuroticlayers. It is among the most dreaded complications faced by surgeons and of greatest concern because of the risk of evisceration, theneed for immediate intervention, and the possibility of repeat dehiscence, surgical wound infection, and incisional hernia formation.- Usually the first sign of an impending wound dehiscence is the discharge of serosanguineous fluid from the wound, but some patientsmay present with sudden evisceration.- Acute wound failure occurs in approximately 1% to 3% of patients who undergo an abdominal operation.- Dehiscency most often develops 7 to 10 days postoperatively but may occur anytime after surgery from 1 to more than 20 days- Surgical site infections most commonly occur 5 to 6 days postoperatively but may develop sooner or later than that but with purulentdischarge.

Correct Answer. a

Copyright © 2014 Delhi Academy of Medical Sciences, All Rights Reserved. 23/86

(49).

The following fluid resuscitation is suitable in first for a 12kg child with8% burns:

a. 46ml of human albumin solution.

b. 384ml per 24 hours for Parkland’s formula of crystalloid.

c. 48ml of normal saline for the first period using Muir and Barclay’sregime.

d. 300ml over 24 hours of human albumin solution (HAS) maintenance.

Solution. (d) 300ml over 24 hours of human albumin solution (HAS)maintenance. Ref: Read the text belowSol:- The zone of coagulative necrosis is nonviable, and thus not reversible,while the zones of hyperaemia and stasis are potentially salvageable.- Early burns management is aimed at preventing the progression of thepotentially viable zones into coagulativenecrosis. These zones were described by Jackson.Jackson’s Burn Wound Model:

Correct Answer. d

Copyright © 2014 Delhi Academy of Medical Sciences, All Rights Reserved. 24/86

(50). In which case lymph nodes are resected prophylactically ?

a. Embryonal rhabdomyosarcoma

b. Liposarcoma

c. Fibrosarcoma

d. Neurofibroma

Solution. (a) Embryonal rhabdomyosarcomaRef.: Devita - 1593Sol :- “Embryonal rhabdomyosarcoma shows highest incidence of lymphatic metastasis, so prophylactic lymphadenopathy is done along withsurgical excision of tumor.”- Embryonal rhabdomyosarcoma is MC type of childhood rhabdomyosarcoma arise mostly in orbit and genitourinary tract.- MC adult rhabdomyosarcoma is alveolar rhabdomyosarcoma.- Age is a very important prognostic factor for embryonal rhabdomyosarcoma, more the age worst is prognosis.

Correct Answer. a

(51). Cock’s peculiar tumor is

a. Cylindroma

b. BCC

c. Infected sebaceous cyst

d. SCC

Solution. (c) Infected sebaceous cystRef:Read the text belowSol:- Cock's peculiar tumour is a sebaceous cyst linked growth.- One form of sebaceous cyst is the trichilemmal cyst; these are thought to arise from the hair follicle epithelium and are most commonlyfound on the scalp.- Usually they are multiple but occasionally there can be a proliferating trichilemmal cyst which grows to a large size and ulcerates.- These can resemble a squamous cell carcinoma (both clinically and histologically) and these ulcerating solitary cysts are called Cock'speculiar tumour.- Chronic inflammation causes the cyst to take the form of a granuloma.- This granuloma mimicks a squamous cell tumor. Hence it is a misnomer.- It is not a tumor but looks like a tumor. The most common sites are the ones where one can find hairs. these are, scalp and scrotum.

Correct Answer. c

Copyright © 2014 Delhi Academy of Medical Sciences, All Rights Reserved. 25/86

(52). Chemotherapeutic agent used in soft tissue sarcoma is/are :

a. Doxorubicin

b. Cisplatin

c. Ifosfamide

d. Both a and c

Solution. (d) Both a and cRef.: Read the text belowSol :- Chemotherapy in soft tissue sarcoma :- Indicationo As an adjuvant to surgery for stage IIIo To induce remission in advanced disease.o Main stay of treatment for Ewing’s primitive neuroectodermal tumor and rhabdomyosarcomao Drugso Most active chemotherapeutic drugs are : Doxorubicin and ifosfamideo Gemcitabine and Dacarbazine also have some activity and is particularly active in patient with leomyosarcoma.o Taxanes have selective activity in angiosarcomao Vincristine, etoposide and irinotecan are effective in rhabdomyosarcoma and Ewing’s sarcoma.o Imatinib mesylate is standard therapy for advanced metastatic GIST.

Correct Answer. d

(53). All electrolyte abnormalities are seen in immediate postoperative period, except

a. Negative Nitrogen balance

b. Hypokalemia

c. Glucose intolerance

d. Hypernatremia

Solution. (d) HypernatremiaRef.: Read the text belowSol :- Acute severe hyponatermia occasionally develops in patients undergoing elective surgery.- In these patients the hyponatremia results from excessive administration of sodium free fluid (e.g. D5%) coupled with post surgicalstimulation of ADH release. Premenopausal women are at greater risk.- Other options are seen in immediate postoperative period.

Correct Answer. d

Copyright © 2014 Delhi Academy of Medical Sciences, All Rights Reserved. 26/86

(54). Adjunctive methods that may facilitate splenectomy for hematologic disease and benefit the patient include each of the following except

a. Polyvalent pneumococcal vaccine

b. Heparinization

c. Platelet transfusion

d. Corticosteroids

e. Immediate splenic artery ligation

Solution. (b) HeparinizationRef: Read the text belowSol :- In dealing with splenomegaly, particularly of the hyperactive spleen which is engorged with blood and consuming platelets and otherblood cells, early splenic artery ligation as soon as exposure permits is important, while venous “autotransfusion” from the enlargedspleen continues as the spleen is mobilized.- At this point, the patient would be able to retain transfused platelets, which would help in hemostasis for the operative dissection.- Prior vaccination of pneumococcal vaccine would protect to some degree from at least some strains of encapsulated gram-positiveorganisms for which splenectomized patients are at risk.- Heparin anticoagulation would be contraindicated, and would make hemostasis more difficult in the patient already likely to bleed,particularly in the large potential space left behind following excision of an enlarged spleen.

Correct Answer. b

(55). The commonest cause of metabolic alkalosis is

a. Cancer stomach

b. Pyloric stenosis

c. Small-bowel obstruction

d. Diuretics

Solution. (b) Pyloric stenosisRef.: Read the text belowSol :- Metabolic Alkalosis is the most common acid base disturbance in the surgical patients usually result from excessive vomiting (as inpyloric stenosis) and excessive administration of sodium bicarbonate during resuscitation.

Correct Answer. b

(56). Which of the following suggests unresectablility of a left upper lobe lung cancer ?

a. hemoptysis

b. Pneumonia

c. Malignant pleural effusion

d. A cough specimen with positive sputum cytology

Solution. (c) Malignant pleural effusionRef: Read the text belowSol :- Pneumonia, hemoptysis and clubbing are all signs that may be related to benign pulmonary disease as well as resectable malignancy,and positive sputum cytology on cough specimen does not rule out resection for cure since is its presumed to originate from the canceralready identified in the left upper lobe.- If the cytology were from a catheter transbronchial aspirant from the right lung, it would suggest bilateral disease making this patientwith clubbing and pneumonia inoperable as well as suggesting a tumor that is nonresectable.- The single contraindication to operation planned for resection for cure is the positive finding of a malignant pleural effusion, since thisindicates that the cancer has spread beyond the confines of the lobe and would not be cured even by left pneumonectomy.

Correct Answer. c

Copyright © 2014 Delhi Academy of Medical Sciences, All Rights Reserved. 27/86

(57). Swirl sign in epidural hematoma suggest?

a. Active bleed in EDH

b. Chronic hemolysed blood

c. Resolved hematoma

d. Acute EDH

Solution. (a) Active bleed in EDHRef: Read the text belowSol:

Correct Answer. a

(58).

Which of the following most common cause of hypotension in fractureribs (T10-T12) ?

a. Abdominal solid visceral organ injury

b. Injury to aorta

c. Inter costal artery damage

d. Pulmonary contusion

Solution. (a) Abdominal solid visceral organ injuryRef.: Read the text belowSol :

Correct Answer. a

Copyright © 2014 Delhi Academy of Medical Sciences, All Rights Reserved. 28/86

(59). The Hunterian–Ligature operation is performed for :

a. Varicose veins

b. Arteriovenous fistulae

c. Aneurysm

d. Aortic dissection

Solution. (c) AneurysmRef.: Read the text belowSol :

Correct Answer. c

(60). Given pulmonary procedure in children is indicated in which of the following conditions ?

a. Large ventricular septal defect

b. Coarctation of the aorta

c. “Pink” tetralogy of fallot

d. Distal pulmonary atresia

Solution. -NA-

Correct Answer. a

Copyright © 2014 Delhi Academy of Medical Sciences, All Rights Reserved. 29/86

(61). The most frequently performed procedure for coronary insufficiency currently is

a. Internal mammary artery bypass

b. Coronary endarterctomy

c. Coronary thrombectomy

d. Aortocoronary bypass graft

Solution. (d) Aortocoronary bypass graftRef:Read the text belowSol:- Thrombectomy for coronary artery thrombosis is as temporary as coronary endarterectomy, both of which do not improve blood flowlong-term and, hence,myocardial performance.- The Vineberg internal mammary artery bypass was useful when originally proposed andremains so for selected indications, but the majority of patients today undergo bypass grafting from aorta to coronary arteries throughreversed saphenous veins or some other conduit than the internal mammary artery.- Percutaneous transcatheter coronary angioplasty might be applicable to patients with isolated lesions, but most coronaryaltherosclerosis is generalized with not just a single focal significant stenotic lesion, so even patients with coronary artery bypass graftfrom aortas, which is the most frequently performed operation.

Correct Answer. d

(62). Which of the following is not a absorbable sutures

a. Catgut

b. Silk

c. Polyglyconate (Maxon)

d. Polyglactin (Vicryl)

Solution. (b) SilkRef:Read the text belowSol:- Silk is non absorbable suture.- Catgut is broken by proteolysis while Vicryl by hydrolysis.- Polyglconate is also absorbable but does not retain enough tensile strength.

Correct Answer. b

(63). Most common presentation of intracranial aneurysm is :

a. Coarctation of aorta

b. Systemic hypertension

c. Hypotension

d. Intracranial haemorrhage

Solution. (d) Intracranial haemorrhageRef.: Harrison - 2531Sol :- “Most of the intracranial aneurysms presents as a sudden intracranial haemorrhage.”- Most common cause of subarachnoid haemorrhage is trauma followed by rupture of Berry or saccular aneurysm in brain.

Correct Answer. d

Copyright © 2014 Delhi Academy of Medical Sciences, All Rights Reserved. 30/86

(64). Andy Gump deformity is best avoided by?

a. Avoiding resection of anterior mandibular arch

b. Avoiding resection of angle of mandible

c. Avoiding resection of coronoid process of mandible

d. Avoiding resection of condyloid process of mandible

Solution. (a) Avoiding resection of anterior mandibular archRef: Read the text belowSol:

Correct Answer. a

Copyright © 2014 Delhi Academy of Medical Sciences, All Rights Reserved. 31/86

(65). First treatment of rupture of varicose veins at the ankle should be :

a. Rest in prone position of patient

b. Application of a tourniquet proximally

c. Application of a tourniquet distally

d. Direct pressure and elevation

Solution. (d) Direct pressure and elevationRef.: Read the text belowSol :

Correct Answer. d

(66). Left sided portal hypertension is best treated by :

a. Splenectomy

b. Portocaval shunt

c. Leno-Renal shunt

d. Spleno-Renal shunt

Solution. (a) SplenectomyRef: Sabistom = p 1273Sol :Portal hypertension secondary to isolated splenic vein thrombosis is called left sided portal hypertension. The principal cause of thisincludes :- Pancreatitis- Pancreatic pseudocyst- Pancreatic neoplasm and Trauma- As blood traverse the stomach, large gastric varices are produced that may rupture and bleed (oesophageal varices are uncommon)- Diagnosis is made by selective splenic arteriography- Treatment is splenectomy.

Correct Answer. a

Copyright © 2014 Delhi Academy of Medical Sciences, All Rights Reserved. 32/86

(67). Regarding “Apathetic thyrotoxicosis” all are true except :

a. Common in elderly

b. Delirium is a feature

c. Atrial fibrillation may be seen

d. Radio iodine is treatment of choice

Solution. (d) Radio iodine is treatment of choiceRef.: Read the text belowSol :- When thyrotoxicosis occurs in the elderly, there is a long asymptomatic phase during which osteoporosis and cardiac abnormalities candevelop unnoticed.- When symptoms finally emerge, they may be limited to weight loss and heart failure complicated by atrial fibrillation.- The goiter itself may not be obvious on physical examination, especially when most of it is behind the sternum.

Correct Answer. d

(68). Estimation of the following hormones is useful while investigating a case of gynecomastia except :

a. Testosterone

b. Prolactin

c. Estradiol

d. Luteinising hormone

Solution. (b) Prolactin Ref.: Read the text below Sol : Diagnostic Evaluation for Gynecomastia- The evaluation of patients with gynecomastia should include :(1) a careful drug history;(2) measurement and examination of the testes (if both are small, a chromosomal karyotype should be obtained; if the testes areasymmetric, a workup for testicular tumor should be instituted);(3) evaluation of liver function; and(4) endocrine workup to include measurement of serum androstenedione or 24-h urinary 17 ketosteroids (usually eveated in feminizingadrenal states), measurement of plasma estradi/ol and hCG (helpful if elevated but usually normal).

Correct Answer. b

(69). Which is not a paraneoplastic syndrome for Hepatocellular Carcinoma

a. Hypercalcemia

b. Hypoglycemia

c. Erythrocytosis

d. Hyperglycemia

Solution. (d) HyperglycemiaRef: Read the text belowSol :- Less than 1% of cases of HCC present as a paraneoplastic syndrome, most commonly hypercalcemia, hypoglycemia, and erythrocytosis

Correct Answer. d

Copyright © 2014 Delhi Academy of Medical Sciences, All Rights Reserved. 33/86

(70). The most common complication see in hiatus hernia is

a. Oesophagitis

b. Aspriation pneumonitis

c. Volvulus

d. Esophageal stricture.

Solution. (a) OesophagitisRef.: Read the text belowSol :- A hiatus hernia per se does not cause any symptoms.- The condition promotes reflux of gastric contents (via its direct and indirect actions on the anti-reflux mechanism) and thus isassociated with gastroesophageal reflux disease (GERD).- In this way a hiatus hernia is associated with all the potential consequences of GERD – heartburn, esophagitis, barrett's esophagus,esophageal cancer and dental erosion most common being esophagitis

Correct Answer. a

(71). Which of the following is not a poor prognostic indicator of breast cancer?

a. S phase 15%.

b. Cathepsin D – High

c. Epidermal growth factor – High

d. Ki 67 – 10%

Solution. (a) S phase 15%.Ref: Read the text belowSol:

Correct Answer. a

Copyright © 2014 Delhi Academy of Medical Sciences, All Rights Reserved. 34/86

(72). Which quadrant of breast has least chance of turning malignant?

a. Upper outer

b. Upper inner

c. Lower outer

d. Lower inner

Solution. (c) Lower outerRef.: Read the text belowSol :- Least chance of turning malignant : Lower inner quadrant- Highest chance of turning malignant : upper outer quadrant

Correct Answer. c

(73). A 30 year old woman comes with dysphagia for both solid and liquids. Barium swallow shows parrot beak appearance.On esophagealmanometry, increased LES pressure.Management includes:

a. Ca channel blockers

b. Botulinum toxin

c. Myotomy

d. All of the following

Solution. (d) All of the followingRef: Read the text belowSol:

Correct Answer. d

Copyright © 2014 Delhi Academy of Medical Sciences, All Rights Reserved. 35/86

(74). Patterson Brown Kelly syndrome is characterized by all except:

a. Lower esophageal web

b. Iron deficiency anemia

c. Common in adult female

d. Premalignant

Solution. (a) Lower esophageal webRef: Read the text belowSol:- Plummer–Vinson syndrome (PVS), also called Paterson–Brown–Kelly syndrome or sideropenic dysphagia, is a rare disease characterizedby difficulty in swallowing, iron deficiency anemia, glossitis, cheilosis and upper esophageal webs.- Treatment with iron supplementation and mechanical widening of the esophagus generally provides an excellent outcome.

Correct Answer. a

(75). Commonest cause of pyogenic liver abscess:

a. Biliary Sepsis

b. Biliary Colic

c. Appendicitis

d. Sigmoid Diverticulitis

Solution. (a) Biliary SepsisRef: Read the text belowSol:- “ The majority of hepatic abscesses were used to result from portal spread of Intra-abdominal infections (e.g. appendicitis, diverticulitis,colitis) but with approved management of these conditions spread now occur primarily through the biliary tree or arterial supply as inpatients suffering from cholangitis or after biliary tract manipulation.”

Correct Answer. a

(76). Anchovy sauce pus is a feature of :

a. Amoebic liver abscess

b. Lung abscess

c. Splenic abscess

d. Pancreatic abscess

Solution. (a) Amoebic liver abscessRef: Read the text belowSol:- Amoebic liver abscess have a scant inflammatory reaction at their margin and a shaggy fibrin lining.- Because of hemorrhage into the cavities the abscesses are filled with a chocolate paste like material similar to anchovy sauce, so calledas anchovy sauce pus.”

Correct Answer. a

Copyright © 2014 Delhi Academy of Medical Sciences, All Rights Reserved. 36/86

(77). A patient presented to emergency ward with massive upper GI bleeding. On examination, he has mild splenomegaly. In the absence ofany other information available which of the following is the most appropriate therapeutic modality?

a. Intravenous Propranolol

b. Intravenous vasopressin

c. Intravenous pantoprazole

d. Intravenous Somatostatin

Solution. (c) Intravenous pantoprazoleRef: Read the text belowSol:- “ Pantoprazole is a proton pump blocker which are included in supportive treatment of hematemesis irrespective of cause.”- By decreasing the gastric acidity it decreases the chance of aspiration pneumonitis.

Correct Answer. c

(78). The most clinically significant early physiologic abnormality in post-traumatic pulmonary insufficiency is

a. Increased physiologic shunt

b. Increased dead space

c. Ventilator barotraumas

d. Cardiogenic pulmonary edema

Solution. (a) Increased physiologic shuntRef:Read the text belowSol:- Shock lung has increased fluid content, much of it interstitial. This increased fluid volume and decreased gas space has lead to agreater fraction of the lung perfused but not ventilated which is by definition the physiologic shunt.- Pulmonary fibrosis occurs at a very late stage and can inhibit gas exchange and impair compliance.- This same alveolar capillary blockade can occur from lung injury by high pressures or oxygen tensions administered to the lung.- However, the physiologic basis for the inability of the lung to oxygenate blood adequately is the greater fraction of lung perfusedwithout effective ventilation, that is, physiologic shunt.

Correct Answer. a

(79). Advantages of crystalloid solution over colloid in fluid resuscitation include all of the following except :

a. Cost

b. Availability

c. Retrievability

d. Edema

Solution. (d) EdemaRef:Read the text belowSol:- Crystalloid solution is readily available at 1/20th of the cost of albumin, and has fewer associated pyrogenic reactions.- It is also possible to retrieve infused crystalloid through diuresis in patients with functioning kidneys.- It does cross permeable membranes to accumulate in “third space” components of extracellular fluid; whereas, albumin has a longercirculating half life in patients tending toward congestive edema.

Correct Answer. d

Copyright © 2014 Delhi Academy of Medical Sciences, All Rights Reserved. 37/86

(80). Ramu, 40-year-old male chronic alcoholic, diagnosed as cirrhosis, presents with a lump in the right lobe of liver. Serum AFP level isnormal most probable diagnosis is :

a. Fibrohyperplasia

b. Hepatocellular carcinoma

c. Secondaries

d. Hepatic adenoma

Solution. (b) Hepatocellular carcinomaRef: Devita - 994Sol :- “AFP level is elevated in 70% of individual with HCC in Asian countries and 30% in European countries.”- Clinical criteria of secondaries in liver :- Both lobes are enlarge.- Nodular growth with umbilication (due to central necrosis) is characteristic.- Sharp border.- Evidence of primary.- Thrill or bruit never occur.

Correct Answer. b

(81). Caudate lobe refers to :

a. Segment I

b. Segment II

c. Segment III

d. Segment IV

Solution. (b) Segment IIRef: Bailey 25/e, p 1081Sol :

Correct Answer. b

Copyright © 2014 Delhi Academy of Medical Sciences, All Rights Reserved. 38/86

(82). Indication of liver transplantation are all of following except :

a. Biliary ateresia

b. Sclerosing cholangitis

c. Hepatitis A

d. Cirrhosis

Solution. (a) Biliary ateresiaRef: Harrison - 1984Sol :

Correct Answer. a

(83). Which of the following characteristics of a patient with thyroid cancer has the most effect on prognosis?

a. Degree of differentiation

b. Tumor stage

c. Lymph node involvement

d. Patient’s age

Solution. (d) Patient’s ageRef:Read the text belowSol:- The biologic factors predominate in prognosis of thyroid cancers and many of them are “given” rather than controllable.- The single most important is the patient’s age, since if all other features were held constant, the younger patient has a better prognosis.- Lymph node involvement, as previously noted, is not a principle factor in changing the staging of tumors of the same size, but size is afactor, and staging is principally determined by T-stage with much less contribution from N-staging.- Differentiation of the tumor for a given cell type, such as papillary, is a significant prognostic factor for groups of patients, but is not assignificant as age.- Since most of the patients have well differentiated tumors, it is also humbling to note that one prognostic factor absent as a principledeterminant of how well patients do is treatment, about which each clinician carries such passionate belief that it is difficult to testtreatment options to determine a superior method which we honestly do not know.

Correct Answer. d

Copyright © 2014 Delhi Academy of Medical Sciences, All Rights Reserved. 39/86

(84). The anatomic division of the liver for right lobar hepatectomy may be defined by each of the following except

a. The falciform ligament

b. The distribution of the right hepatic artery

c. The distribution of the right portal vein

d. The drainage area of the right hepatic veins

Solution. (a) The falciform ligamentRef: Read the text belowSol :- The anatomic division of the liver is very important for the three part blood flow which is an important consideration in hepaticresection. The neat anatomic division of the falciform ligament actually separates the left lateral lobe of the liver from the mediansuperior and median inferior components of the left lobe and it is not the true dividing line for hepatic lobectomy.- The right and left lobes are divided through an anatomic line that would pass through the gallbladder fossa to the vena cava, whichcorrelates with the distribution of the right hepatic artery, portal and venous systems.

Correct Answer. a

(85). Which of the following group constitute frey’s syndrome-

a. Hyperhydrosis, enophthalmos and miosis

b. Anhidrosis, enophthalmos and miosis

c. Redness and sweating over the auriculotemporal during meal

d. Pain over the distribution of the auriculotemporal nerve during meal

Solution. (c) Redness and sweating over the auriculotemporal during mealRef: Read the text belowSol :- Frey's syndrome (also known as Baillarger’s syndrome, Dupuy’s syndrome, Auriculotemporal syndrome or Frey-Baillarger syndrome) isa food related syndrome which can be congenital or acquired specially after parotid surgery and can persist for life.- The symptoms of Frey's syndrome are redness and sweating on the cheek area adjacent to the ear.- They can appear when the affected person eats, sees, dreams, thinks about or talks about certain kinds of food which produce strongsalivation

Correct Answer. c

Copyright © 2014 Delhi Academy of Medical Sciences, All Rights Reserved. 40/86

(86). Which of the following hernias follows the path of the spermatic cord within the cremaster muscle?

a. Femoral

b. Direct inguinal

c. Indirect inguinal

d. Spigelian

Solution. (c) Indirect inguinalRef: Read the text belowSol:- An indirect inguinal hernia leaves the abdominal cavity by entering the dilated internal inguinal ring and passing along theanteromedial aspect of the spermatic cord.- The internal inguinal ring is an opening in the transversalis fascia for the passage of the spermatic cord; an indirect inguinal hernia,therefore, lies within the fibers of the cremaster muscle.- Repair consists of removing the hernia sac and tightening the internal inguinal ring. A femoral hernia passes directly beneath theinguinal ligament at a point medial to the femoral vessels, and a direct inguinal hernia passes through a weakness in the floor of theinguinal canal medial to the inferior epigastric artery.- Each is dependent on defects in Hesselbach’s triangle of transversalis fascia and neither lies within the cremaster muscle fibers. Repairconsists of reconstructing the floor of the inguinal canal.- Spigelian hernias, which are rare, protrude through an anatomic defect that can occur along the lateral border of the rectus muscle atits junction with the linea semilunaris. An interparietal hernia is one in which the hernia sac, instead of protruding in the usual fashion,makes its way between the fascial layers of the abdominal wall.- These unusual hernias may be preperitoneal (between the peritoneum and transversalis fascia), interstitial (between muscle layers), orsuperficial (between the external oblique aponeurosis and the skin).

Correct Answer. c

(87). The most common hernia in females is:

a. Direct inguinal hernia

b. Indirect inguinal hernia

c. Femoral hernia

d. Spigelian hernia

Solution. (b) Indirect inguinal herniaRef.: Read the text belowSol :- Most common type of hernia in men: Indirect inguinal hernia- Femoral hernia is more common in women, but it is not the most common type of hernia in females.- Most common type of hernia in women: Indirect inguinal hernia- Both indirect as well as femoral hernia are more common on the right side- Direct inguinal hernias have no side predilection. They are more common in elderly.

Correct Answer. b

Copyright © 2014 Delhi Academy of Medical Sciences, All Rights Reserved. 41/86

(88). Van Nuys score is used for

a. DCIS

b. LCIS

c. Infiltrating Ductal Cancer

d. Infiltrating Lobular Cancer

Solution. (a) DCISRef: Read the text belowSol:

Correct Answer. a

(89). Lyphatic from left upper quadrant of breast drain into all of the following groups of lymph nodes except:

a. Anterior axillary

b. Central

c. Apical

d. Parasternal

Solution. (d) ParasternalRef.: Read the text belowSol :Lymphatic Drainage of the Breast :- Lymph from the nipple, areola, and lobules of the mammary glands drains to a subareolar lymphatic plexus.- From there, a system of interconnecting lymphatic channels drains lymph to various lymph nodes.- The majority of the lymph, especially from the lateral quadrants of the breast, drains to the external mammary (anterior, pectoral)group of lymph nodes, and from there to the axillary nodes.- Some lymph vessels may travel directly to the subscapular (posterior, scapular) group of lymph nodes- From the upper part of the breast, a few lymph vessels pass directly to the subclavicular (apical) group of lymph nodes.- The axillary lymph nodes usually receive > 75% of the lymph drainage from the breast.- The rest is derived primarily from the medial aspect of the breast, flows through the lymph vessels that accompany the perforatingbranches of the internal mammary artery, and enters the parasternal (internal mammary) group of lymph nodes.

Correct Answer. d

Copyright © 2014 Delhi Academy of Medical Sciences, All Rights Reserved. 42/86

(90). Management of pulmonary complications of tuberculosis refractory to antibiotic management include each of the following except

a. Decortication

b. Chest wall reconstruction

c. Hyperbaric oxygen therapy

d. Bronchoscopic brush biopsy and lavage

Solution. (c) Hyperbaric oxygen therapyRef: Read the text belowSol :- Tuberculosis can result in local destruction of affected lung with cavitation. If this cavitary destruction is large, if it is multiple in onelobe, and if it is not successfully drained intrabronchially through the assistance of bronchoscopic brush biopsy to enter the cavity andlavage, lobectomy may be advised.- A thickened pleura is likely, particularly if a secondary bacterial infection and empyema result which may require thorough drainage ofthe pleural space and decortications of the encased lung in order to allow expansion.- If that expansion is not possible with the lung coming up to chest wall, the chest wall is actually collapsed down onto the lung, in a typeof chest wall reconstruction called thoracoplasty.- Each of these surgical methods has been used with success with management of complications of tuberculosis.- Hyperbaric oxygen is not successful in managing these complications.

Correct Answer. c

(91). One method of pre-operative therapy that has been helpful in facilitating liver resection for tumor has been

a. Pre-operative external beam radiation

b. Arteriographic vascular occlusion

c. Portal venous thrombosis

d. Intensive cancer chemotherapy with leucovorin rescue

Solution. (b) Arteriographic vascular occlusionRef: Read the text belowSol :- Hepatoma is not a radiosensitive tumor and no pre-operative utility would be expected from radiotherapy or intensive cytotoxicchemotherapy, since it is alos highly unresponsive to drug therapy.- Immunotherapy and nutritional therapy might make a marginal difference in the host response to operation, but are inadequate tomake any marginal difference that would be worth a delay.- The one technique listed that has been proven effective in selective instances is devascularization of the liver segment anticipated to beresected, which gives a head-start on the regeneration of liver function in adjacent liver to be preserved.- It is also suggested to decrease the tumor neovascularity involved in the tumor-bearing segment, but this is difficult to judgeprospectively and impossible to control in comparative trials.- Radiographic occlusion is nonetheless and advocated for some selected pre-operative hepatoma management.

Correct Answer. b

Copyright © 2014 Delhi Academy of Medical Sciences, All Rights Reserved. 43/86

(92). Of the following transmissible agents that may give rise to focal liver infections, which may cause an anaphylactic response if thecontents are spilled in the event of resection or rupture ?

a. Entamoeba histolytica

b. Staphylococcus aureus

c. Clonorchis sinensis

d. Echinococcus granulosus

Solution. (d) Echinococcus granulosusRef: Read the text belowSol :- Pyogenic abscesses, such as that with Staphylococcus or Proteus may cause febrile reactions and can lead to septicemia, but nothypersensitivity reactions.- Clonorchis sinensis is an Oriental liver fluke that takes up residence in the biliary tree, and is a source of inflammatory response, butnot anaphylaxis. Amoeba are protozoans that in some zymogen classes not only give rise to colonic inflammation but invade the portalblood stream and can set up hepatic abscess, which when drained does not give anaphylaxis.- This is not the case with hydatid cyst caused by Echinococcus granulosus. Not only can rupture of the cyst result in implantation anddissemination of daughter cysts, Echinococcus can cause a lethal hypersensitivity reaction as well if spilled into the peritoneum.

Correct Answer. d

(93). Which of these surgery is not used as a limb reduction procedure for lymphoedema ?

a. Homans procedure

b. Sistrunk procedure

c. Thompson procedure

d. Hill procedure

Solution. (d) Hill procedureRef– Read the text belowSol:- Hill procedure indicated for GERD.- Limb reduction procedures for lympho-edema are:- Homans procedure- Kondolean Procedure- Miller/Sistrunk staged excision operation- Sistrunk procedure- Thompson procedure- Charles procedure- The Buck Fascia Procedure

Correct Answer. d

(94). The axillary nodes receive how much amount of lymphatic drainage of the breast?

a. 25%

b. 30%

c. 70%

d. 85%

Solution. (d) 85%Ref– Read the text belowSol:- Lymphatic of the breast drain predominantly into the axillary and internal mammary lymph nodes.- The axillary nodes receive approximately 85% of the drainage.

Correct Answer. d

Copyright © 2014 Delhi Academy of Medical Sciences, All Rights Reserved. 44/86

(95). Poland syndrome includes congenital absence of the breast along with:

a. Absence of sternal portion of the pectoralis major

b. Deafness

c. Glomerular basement membrane disease

d. Alopecia

Solution. (a) Absence of sternal portion of the pectoralis majorRef– Read the text belowSol:- Poland syndrome (also Poland's syndrome, Poland's syndactyly, Poland sequence, and Poland's anomaly) is a rare birth defectcharacterized by underdevelopment or absence of the chest muscle (pectoralis) on one side of the body and (but not always) webbing ofthe fingers (cutaneous syndactyly) of the hand on the same side (ipsilateral hand) mostly common on the right side of body and foundmore in males than females.

Correct Answer. a

(96). Most common tumour of the chest wall is:

a. Chondroma

b. Lipoma

c. Fibroma

d. Rhabdomyoma

Solution. (a) ChondromaRef– Read the text belowSol:- The most common tumour of chest wall is that of the rib (chondroma or osteoma).

Correct Answer. a

(97). Each of the following conditions is an indication for splenectomy except :

a. Thrombocytopenia

b. Traumatic splenic laceration in a child with minimum blood loss

c. A splenic artery aneurysm within the splenic pulp

d. Splenic abscess

Solution. (b) Traumatic splenic laceration in a child with minimum blood lossRef: Read the text belowSol :- Thrombocytopenia from idiopathic or other acquired origins may respond to splenectomy if steroid therapy no longer induces aresponse in raising platelets. Spherocytosis is a frequent hemolytic indication for splenectomy as one of the only treatments and withexcellent success rate.- Splenic abscess and splenic artery aneurysm both are treated by splenectomy if simple drainage is inadequate or an exterior splenicartery aneurysmectomy is not possible. In children, however, splenectomy is to be avoided for trauma unless there is hemodynamicinstability and no other way to control the intra-abdominal hemorrhage.- With minimal blood loss and stable vital signs, minimal splenic disruption can be managed by observation in order that the splenicfiltration and immunologic function be preserved. If surgical exploration is undertaken, attempts at repair of minor traumatic splenicinjuries are recommended, particularly in children, and splenectomy reserved for unsalvageable disruption or patients with multipleother injuries requiring urgent attention.

Correct Answer. b

Copyright © 2014 Delhi Academy of Medical Sciences, All Rights Reserved. 45/86

(98). A 32-year-old diabetic woman who has taken contraceptive pills for 12 years develops RUQ pain. CT scan of the abdomen reveals a 5-cmhypodense lesion in the right lobe of the liver consistent with a hepatic adenoma. What should the patient be advised to do?

a. Undergo excision of the adenoma

b. Stop oral contraceptives only

c. Undergo right hepatectomy

d. Have serial CT scans every 6 months

Solution. (a) Undergo excision of the adenomaRef: Read the text belowSol :- Hepatic adenomas are associated with an increased incidence in patients receiving oral contraceptives, diabetes, and pregnancy.- Most patients are symptomatic with pain and bleeding.- Because of the real risk of intraperitoneal or intratumoral bleeding as well as malignant transformation, excision of the adenoma isrecommended.- Tumors are removed by enucleation or with a narrow rim of normal parenchyma, and major liver resection is not required

Correct Answer. a

(99). A 35-year-old woman presents with focal nodular hyperplasia. This condition is similar to hepatic adenoma, in that it does what?

a. Frequently causes symptoms

b. Tends to lead to liver rupture

c. LFT and alpha fetoprotein (AFP) are normal

d. Easily detected by CT scan of the liver

Solution. (c) LFT and alpha fetoprotein (AFP) are normalRef: Read the text belowSol :- Unlike hepatic adenomas, these lesions do not usually cause symptoms.- Unlike hepatic adenomas, focal nodular hyperplasia does not tend to cause intramural bleeding with rupture into the peritoneal cavity.- CT or US scan may frequently miss the lesion, because it is so dense. There is no definite relationship with oral contraceptives.- Focal nodular hyperplasia lesions are not well encapsulated and have a central stellate scar.- Malignant changes have not been reported. LFT and AFP are normal in both conditions.

Correct Answer. c

(100). In discussing the treatment of a 42-year-old man with severe liver cirrhosis, the possibility of heterotopic transplantation is considered.Which statement about heterotopic liver transplantation is true?

a. It implies removal of the recipient’s liver.

b. It is preferable to orthotopic liver transplantation.

c. It should be done in the iliac vessels.

d. It is rarely associated with long-term survival.

Solution. (d) It is rarely associated with long-term survival.Ref: Read the text belowSol :- Is rarely associated with long-term survival.- Heterotopic (to a remote position) auxiliary transplantation is only occasionally indicated where orthotopic transplantation cannot becarried out.- Long-term survival with this procedure is limited (2 of 69 cases).- Hetrotopic auxiliary liver transplants require low outflow pressure and are, therefore, most likely to succeed if placed proximally asclose to the heart as possible.- One advantage of this procedure is that the procedure is technically easier, because the patient’s liver is not disturbed.

Correct Answer. d

Copyright © 2014 Delhi Academy of Medical Sciences, All Rights Reserved. 46/86

(101). Splenectomy is often indicated in the management of which of the following?

a. Hereditary spherocytosis

b. Hereditary neurofibromatosis

c. Aplastic anemia

d. Pheochromocytoma

Solution. (a) Hereditary spherocytosisRef: Read the text belowSol :- In hereditary spherocytosis, the abnormally shaped erythrocytes fail to pass through the splenic pulp and are more prone to earlierdestruction.- In hereditary elliptocytosis, the erythrocyte membrane also is abnormal. Children with spherocytosis should undergo splenectomyaround their fourth birthday.- Other less common hematological indications for splenectomy are thalassemia, sickle cell anemia, autoimmune anemia, and an enlargedspleen that becomes a major site of red cell sequestration.

Correct Answer. a

(102). A 2-year-old African-American boy is diagnosed as having hereditary spherocytosis. His parents should be informed that this condition iswhich of the following?

a. It is not associated with a marked increase in gallstones.

b. It is transmitted as a recessive trait.

c. It is diagnosed by showing RBCs undergo lysis at a higher osmotic pressure.

d. It is characterized by a low reticulocyte count.

Solution. (c) It is diagnosed by showing RBCs undergo lysis at a higher osmotic pressure.Ref: Read the text belowSol :- Characterized by RBCs that undergo lysis at a higher osmotic pressure. Gallstones are frequently encountered as a result of increasedproduction of bilirubin.- Hereditary spherocytosis is transmitted as an autosomal-dominant trait.- Because of a fault in the RBC membrane, the cells are smaller and round and undergo lysis in a minor vessel, which results in a relativeobstruction to flow.

Correct Answer. c

(103). Drug used for sclerotherapy of varicose veins are:

a. Ethanolamine Oleate

b. Polidocanol

c. Ethanol

d. All of the above

Solution. (d) All of the aboveRef: Bailey and Love’s Surgery--923Sol:- Sclerotherapy remains the primary treatment for small-vessel varicose disease of the lower extremities.- These small vessels include telangiectasias, venulectasias, and reticular ectasias.- Telangiectasias the flat red vessels smaller than 1 mm in diameter. Venulectasias are blue, sometimes distended above the skin surface,and smaller than 2 mm in diameter. Reticular veins have a cyanotic hue and are 2-4 mm in diameter.- Large varicosities do not respond as well as small varicosities to sclerotherapy. Sclerosants include the following :1. Detergent-Disrupt vein cellular membrane (protein theft denaturation) (i) Sodium tetradecyl sulfate (ii) Polidocanol (iii) Soldiummorrhuate (iv) Ethanolamine Oleate 2. Osmotic agents – Damage the cell by shifting the water balance through cellular gradient(osmotic) dehydration and cell membrane denaturation (i) Hypertonic sodium chloride solution (ii) Sodium chloride solution withdextrose 3. Chemical irritants – Damage the cell wall by direct caustic destruction of endothelium (i) Chromated glycerin (ii)Polyiodinated iodine

Correct Answer. d

Copyright © 2014 Delhi Academy of Medical Sciences, All Rights Reserved. 47/86

(104). Which of the following describes Richter’s hernia :

a. The strangulated loop of ‘W’ lies within the abdomen

b. Hernia occurring at the level of arcuate line

c. Involves only a portion of the circumference of the bowel.

d. Hernia occurring through the linea alba

Solution. (c) Involves only a portion of the circumference of the bowel.Ref: Bailey and Love’s Surgery--970Sol:- The Richter hernia occurs when only the antimesenteric border of the bowel herniates through the fascial defect.- The Richter hernia involves only a portion of the circumference of the bowel. As such, the bowel may not be obstructed, even if thehernia is incarcerated or strangulated, and the patient may not present with vomiting.- The Richter hernia can occur with any of the various abdominal hernias and is particularly dangerous, as a portion of strangulatedbowel may be reduced unknowing into the abdominal cavity, leading to perforation and peritionitis.- The strangulated loop of ‘W’ lies within the abdomen, so that local tenderness over the hernia is not marked is a feature of Maydl’shernia.

Correct Answer. c

(105). During thyroidectomy, inferior thyroid artery is ligated at :

a. Maximally away from the gland

b. Close to the gland

c. Halfway from the gland

d. None of the above

Solution. (d) None of the aboveRef: Bailey and Love’s Surgery--989Sol:Devascularization(i) Superior thyroid artery – It splays over the upper pole and hence ligated individually(ii) Inferior thyroid artery are not routinely ligated so protecting the supply to the parathyroids

Correct Answer. d

(106). A67-year-old man is admitted to hospital with a diagnosis of polycythemia vera. He has considerable back pain and is diagnosed ashaving myeloid metaplasia. This condition is characterized by which of the following?

a. Decrease of the connective tissue in the spleen

b. Decrease in the blood elements of the spleen

c. Deterioration after splenectomy

d. A favorable response to alkylating agents

Solution. (d) A favorable response to alkylating agentsRef: Read the text belowSol :- Alkalating agents must be given cautiously, because patients with myeloid metaplasia are sensitive to these agents.- The connective and hemopoietic tissues in the spleen and liver are increased.- Polycythemia vera, myelogenous leukemia, and idiopathic thrombocytosis must be excluded. Splenectomy is often of value.

Correct Answer. d

Copyright © 2014 Delhi Academy of Medical Sciences, All Rights Reserved. 48/86

(107). Hypertension is seen in all except-

a. Adrenal tumors

b. Pheochromocytoma

c. Conn’s syndrome

d. Neuroblastoma

Solution. (c) Conn’s syndromeRef: Read the text belowSol:- Conn’s syndrome is due to hyperaldosteronism secondary to aldosterone producing adrenal adenoma.

Correct Answer. c

(108). Ramu, a 40 year-old male, a chronic smoker presents with claudication and a medial leg ulcer. For the past one month he is having restpain.Which of the following procedures would not relieve his rest pain?

a. Lumbar sympathectomy

b. Omentoplaxy

c. Conservative amputation

d. Femoropopliteal by pass.

Solution. (d) Femoropopliteal by pass.Ref: Read the text belowSol:- This is a case of Burger’s disease (also known as thromboangiitis obliterans), femoropoplitial bypass is of no use in such cases.

Correct Answer. d

(109). Which of the following is true about cavernous Hemangioma:

a. Seen after 3 month of life and is progressive

b. If multiple can lead to CHF as complication

c. Arise from veins of lower limb

d. Surgery is treatment of choice

Solution. (b) If multiple can lead to CHF as complicationRef: Read the text belowSol:Cavernous Hemangioma- Benign vascular neoplasm that arise soon after birth.- Present as bright red or purple swelling arising from vein.- MC site – face, cheek, ears- Non-pulsatile but compressible- Histologically characterized by large blood filled space lined by normal appearing endothelial cells (capillary hemagioma are lined byfetal endothelial cells)- They do not show any tendency to involute, but it may become larger and more troublesome.

Correct Answer. b

Copyright © 2014 Delhi Academy of Medical Sciences, All Rights Reserved. 49/86

(110). A 48-year-old patient presents with dysphagia to both liquids and solids and regurgitation for 3 months. The dysphagia was non-progressive. What is the most likely diagnosis?

a. Carcinoma esophagus

b. Lower esophageal mucosal ring

c. Achalasia cardia

d. Reflux esophagitis with esophageal stricture.

Solution. (c) Achalasia cardiaRef: Read the text belowSol:Dysphagia of achalasia is progressive initially then reaches a plateau phase in most cases- Dysphagia of carcinoma and stricture is clearly progressive.- Regurgitation is also pointing towards achalasia

Correct Answer. c

(111). A 24-year-old woman with rheumatoid arthritis involving the sacroiliac joint and fingers is noted to have splenomegaly and neutropenia(Felty’s syndrome). She is advised to have splenectomy, but she should be informed that

a. Large-joint disease symptoms will lessen.

b. Small-joint disease symptoms will lessen.

c. Neutropenia responds to splenectomy.

d. The joint symptoms will become worse.

Solution. (c) Neutropenia responds to splenectomyRef: Read the text belowSol :- Felty’s syndrome is characterized by splenomegaly, neutropenia, and rheumatoid arthritis. Steroids are used initially, but their effectusually is transient.- Splenectomy favorably alters the leukocyte count; it does not alter the clinical course of rheumatoid arthritis.- As with all patients undergoing elective splenectomy, this patient must be given pneumovax, as well as haemophilus andmeningiococcal vaccines before surgery

Correct Answer. c

(112). A 65-year-old man undergoes cardiac surgery for triple vessel coronary artery disease. What can he anticipate?

a. 95% chance his grafts will occlude after 12 months.

b. 5% chance of living for 5 years.

c. If the internal mammary artery is used as a conduit, patency is increased.

d. Mortality if 10–20% in most centers.

Solution. (c) If the internal mammary artery is used as a conduit, patency is increased.Ref: Read the text belowSol :- The internal thoracic artery is the conduit of choice especially for grafting the left anterior descending (LAD) artery.- Arterial and venous grafts 95% of the time do not occlude after 12.- Seventy-five percent of patients under coronary artery bypass graft (CABG) survive 5 years. Mortality is 2% or lower in most centers.

Correct Answer. c

Copyright © 2014 Delhi Academy of Medical Sciences, All Rights Reserved. 50/86

(113). Three months after aortic valve replacement with a mechanical prosthesis, a 60-year-old man describes malaise, and increasingshortness of breath. Examination reveals pulsus paradoxus. ECG shows low voltage precordially. What test is most useful for making thediagnosis?

a. Stress thallium exam

b. Computer Tomography (CT) examination of chest

c. Echocardiography

d. Serum creatinine phosphokinase (CPK)

Solution. (c) EchocardiographyRef: Read the text belowSol :- This patient has a pericardial effusion. Echocardiography is the most useful in making the diagnosis. CAT scan of the chest can be usedbut is not the best exam.- The other choices do not apply.

Correct Answer. c

(114). Most common surgical complication of enteric fever is :

a. Intestinal perforation

b. Intestinal bleeding

c. Stricture formation

d. Peritonitis

Solution. (b) Intestinal bleedingRef.: Read the text belowSol :- The following are the common complications of thyphoid which the surgeon is concerned with :- Intestinal hemorrhage occurs in 10% 20% of hospitalized patients. This occurs from multiple ulcers and the bowel is extremely friable.- Usually blood transfusion with anti-typhoid drugs cures this condition.- Only in rare cases a Laparotomy may be needed to control life threatening hemorrhage.

Correct Answer. b

(115). Death occurs in what percentage of untreated aortic aneurism cases in 1 year.

a. 50%

b. 70%

c. 80%

d. 100%

Solution. (c) 80%Ref.: Read the text belowSol :- 80% dead in 1 year without surgery.- With surgery, 80% alive in 1 year.- Mortality more patients with hypertension, lung, heart or renal disease. Mortality more if patient > 80 years old.- Operate if fit, mortality < 5%

Correct Answer. c

Copyright © 2014 Delhi Academy of Medical Sciences, All Rights Reserved. 51/86

(116). The preferred treatment for 2.5 cm papillary cancer of right lobe of 30 year old female is?

a. Enucleation

b. Lobectomy and left sided lymph node dissection.

c. Total thyroidectomy and central lymph node dissection.

d. Total thyroidectomy.

Solution. (d) Total thyroidectomy.Ref: Sabistom 18/e, p 1273Sol :- The primary treatment of PTC is surgical ablation. For lesions smaller than 1 cm, there is general agreement in the literature thatlobectomy plus isthmectomy is appropriate, particularly for incidentally found papillary carcinomas.- Younger patients, especially those 15 years or younger, have a high rate of cervical metastasis to the extent that perhaps 90% ofchildren with papillary carcinoma may have documented metastatic activity within the lymph nodes. Therefore, there is consensus thatpatients in this age group undergo total thyroidectomy and lymph node dissection in the presence of palpable cervical lymph nodes.- Additionally, in older patients with a history of neck irradiation, a more aggressive approach may be taken, including totalthyroidectomy and modified neck dissection in those with palpable cervical lymph nodes. There appears to be no survival benefit withprophylactic lymph node dissection for nonpalpable nodes.- Controversy exists about the use of total thyroidectomy versus lobectomy and isthmectomy in adults with a 1- to 2-cm PTC. Theadvantages of total thyroidectomy include the efficient use of radioiodine postoperative treatment.- If papillary carcinoma is manifested as a palpable lesion larger than 2 cm, total thyroidectomy is definitely considered.

Correct Answer. d

(117). Which of the following is not a feature of primary hyperparathyroidism?

a. Hypercalcemia.

b. Hyperphosphatemia.

c. Hyperchloremia.

d. Increased Alkaline phosphatase.

Solution. (b) Hyperphosphatemia.Ref: Sabistom 18/e, p 1273Sol :- It causes hypophosphatemia.- Hyperphosphatemiab is seen in secondary hyperthyroidism.

Correct Answer. b

Copyright © 2014 Delhi Academy of Medical Sciences, All Rights Reserved. 52/86

(118). Which of the following is the most common etiology of adrenal incidentuloma?

a. Early adrenocortical cancer.

b. Adrenal adenoma.

c. Myolipoma.

d. Adrenal Hemorrhage.

Solution. (b) Adrenal adenoma.Ref: Sabistom 18/e, p 1273Sol :

Correct Answer. b

(119). The classic findings of cardiac tamponade are :

a. Hypotension, elevation of jugular venous pressure with a prominent x descent

b. Hypertension, elevation of jugular venous pressure with a prominent x descent

c. Hypotension, elevation of jugular venous pressure with a prominent y descent

d. Hypertension, elevation of jugular venous pressure with a prominent y descent

Solution. (a) Hypotension, elevation of jugular venous pressure with a prominent x descent Reference – Harrison-1416Sol:- The three principal features of cardiac tamponade are elevation of intracardiac- pressures, limitation of ventricular filling, and reduction of cardiac output.- The classic findings of falling arterial pressure, rising venous pressure, and faint heart sounds usually occur only with severe, acutetamponade, as occurs with cardiac trauma or rupture.- Tamponade should be considered in any patient with hypotension and elevation- of jugular venous pressure with a prominent x descent and a diminutive or absent y descent.- In contrast, in constrictive pericarditis, the y descent is prominent- A positive Kussmaul sign is rare in cardiac tamponade, as is a pericardial knock.

Correct Answer. a

Copyright © 2014 Delhi Academy of Medical Sciences, All Rights Reserved. 53/86

(120). A 60 year old man was hit by a car as he crossed the road. He has sustained injuries to his head and both lower legs. One hour after theinjury, he is drowsy and will not open his eyes. He extends his limbs and makes noises to painful stimuli. His right pupil is fixed anddilated. Pulse is 60/mn and BP is 180/120 mmHg.Next step in management would be.

a. Immediate surgery.

b. Urgent CT scan.

c. I.V dexamethasone..

d. I.V mannitol.

Solution. (a) Immediate surgery.Reference – Read the text belowSol:- The clinical scenario is very suggestive of raised intracranial pressure secondary to an extradural haematoma.- Clearly there is not time to arrange diagnostic investigations or definitive treatment. A burr hole might be life saving if the diagnosis iscorrect.

Correct Answer. a

(121). Normal length of common bile duct is?

a. 7 cm long

b. 5 cm long

c. 3 cm long

d. 2 cm long

Solution. (a) 7 cm longRef.: Read the text belowSol :- The common bile duct is formed near the porta hepatitis, by the junction of the cystic and common hepatic ducts. It is usually between 6and 8 cm long.- Its diameter tends to increase somewhat with age but is usually around 6 mm in adult.- The cystic duct drains the gallbladder into the common bile duct. It is between 3 and 4 cm long.

Correct Answer. a

(122). All predisposes to oral cancer except:

a. Erythroplakia

b. Leucoplakia

c. Submucosal fibrosis

d. Lichen planus

Solution. (d) Lichen planusRef.: Bailey - 735Sol :- Out of the given options all except lichen planus are definitely associated with oral cancer.- “If there is an association of oral cancer and lichen planus, the relationship exist only with atrophic or erosive lichen planus”.- All patients with erosive or atrophic lichen planus should be observed and erosive disease should be treated with topical steroids orsystemic steroids in severe cases.

Correct Answer. d

Copyright © 2014 Delhi Academy of Medical Sciences, All Rights Reserved. 54/86

(123). Which of the following statements is true concerning aortocoronary bypass grafting?

a. It is indicated for crescendo (preinfarction) angina

b. It is indicated for congestive heart failure

c. It is not indicated for chronic disabling angina

d. It is associated with a 10% operative mortality

Solution. (a) It is indicated for crescendo (preinfarction) angina References:Read the text below. Sol:- Coronary artery bypass surgery was developed in the late 1960s and is now being regularly performed.- Indications for surgery include chronic disabling angina and crescendo (or preinfarction) angina.- Cardiac catheterization with selective coronary angiography defines the extent of disease, which generally is localized to the proximalsegments of the vessels. Operative mortality is about 2%, and relief of angina is obtained in most affected patients.- Patients with left main coronary artery disease as well as those with triple vessel disease and ventricular dysfunction have an increasedlongevity following successful bypass.- Data regarding extension of life in other groups is conflicting. Coronary artery bypass is not indicated for congestive heart failureunless this condition is ischemic in origin and angiography identifies disease amenable to surgical revascularization.

Correct Answer. a

(124). Which of the following statements is true regarding the thoracic outlet syndrome?

a. It is associated with cervical spine disk disease

b. It is reliably diagnosed by positional obliteration of the radial pulse

c. If conservative measures fail, it is best treated by surgical decompression of the brachial plexus

d. It most commonly affects the median nerve

Solution. (c) If conservative measures fail, it is best treated by surgical decompression of the brachial plexusReferences: Read the text below.Sol:- The thoracic outlet syndrome designates a symptom complex whose precise cause is unknown.- It is felt to result from compression of the brachial plexus or subclavian vessels, or both, in the anatomic space bounded by the first rib,the clavicle, and scalene muscles. Since objective determinants of disease may be lacking or imprecise, the diagnosis often is establishedby resectional surgery.- Carpal tunnel syndrome (compression of the median nerve as it passes through the carpal tunnel of the wrist) and cervical disk diseaseare the two entities most commonly confused with the thoracic outlet syndrome, whose symptoms and signs include pain, paresthesias,edema, venous congestion, and digital vasospastic changes.- Positional dampening or obliteration of the radial pulse is an unreliable finding since it is present in up to 70% of the normal population.Neurologic abnormalities may be documented by nerve conduction studies. Angiographic studies are often negative.- Conservative management, which generally should precede surgery, consists of an exercise program to strengthen shoulder girdlemuscles and decrease shoulder droop.- Operative treatment includes division of the scalenus anticus and medius muscles, first rib resection, cervical rib resection, or acombination of all three.

Correct Answer. c

Copyright © 2014 Delhi Academy of Medical Sciences, All Rights Reserved. 55/86

(125). Gum tumor with 2 contra lateral mobile lymph node in cheek comes under all of the following except:

a. T3 N2 M0

b. T2 N2 M0

c. T4 N2 M0

d. T3 N3 M0

Solution. (d) T3 N3 M0Ref.: CSDT - 264Sol :

Correct Answer. d

(126). Thyroglossal fistula develops to –

a. Developmental anomaly

b. Injury

c. Incomplete removal of thyroglossal cyst

d. Inflammatory disorder

Solution. (c) Incomplete removal of thyroglossal cystRef.: Read the text belowSol :Thyroglossal fistula- It is never congenital- It follows infection as inadequate removal of thyroglossal cyst.- Treatment :As thyroglossal tract is closely related to the body of hyoid bone, the central part must be excised together with the cyst or fistula(Sistrunk operation).

Correct Answer. c

Copyright © 2014 Delhi Academy of Medical Sciences, All Rights Reserved. 56/86

(127).

Which one of the following is the most common site of Berryaneurysm ?

a. Vertebrobasilar artery

b. Origin of posterior communication artery

c. Anterior communicating artery

d. Anterior choroidal artery

Solution. (c) Anterior communicating arteryRef.: Read the text belowSol :

Correct Answer. c

(128). Steering wheel injury on chest of a young man reveals multiple fractures of ribs and paradoxical movement with severe respiratorydistress. X-ray shows pulmonary contusion on right side without pneumothorax or haemotho-rax. What is the initial treatment of choice?

a. Immediate internal fixation

b. Endotracheal intubation and mechanical ventilation

c. Thoracic epidural analgesia and O2 therapy

d. Stabilization with towel clips

Solution. (b) Endotracheal intubation and mechanical ventilationRef.: Bailey and Love’s Short Practice of Surgery, (24the Ed.), Pg 868Sol :- The treatment of severe flail chest is endotracheal intubation and positive pressure ventilation for 3 weeks until flail segment stabilizes.- Thoracotomy and fracture fixation is required occasionally if there is underlying lung injury and it has to be treated at same time.

Correct Answer. b

Copyright © 2014 Delhi Academy of Medical Sciences, All Rights Reserved. 57/86

(129). All of the following are associated with thyroid storm, except:

a. Surgery for thyroiditis

b. Surgery for thyrotoxicosis

c. Stressful illness in thyrotoxicosis

d. I131 therapy for thyrotoxicosis

Solution. (a) Surgery for thyroiditisRef.: Bailey - 791Sol :

Correct Answer. a

Copyright © 2014 Delhi Academy of Medical Sciences, All Rights Reserved. 58/86

(130). Which of the following is not done in case of pheochromocytoma?

a. FNAC

b. MRI

c. CI

d. MIBG

Solution. (a) FNACRef.: Harrison - 2151Sol :- Percutaneous fine-needle aspiration of chromaffin tumors is contraindicated; indeed, pheochromocytoma should be excluded before adrenallesions are aspirated.

Correct Answer. a

(131). Dissection of which artery is seen in pregnancy :

a. Carotid artery

b. Aorta

c. Coronary A

d. Femoral artery

Solution. (b) AortaRef.: Harrison - 1565Sol :- The most common artery dissection seen in pregnancy is aortic dissection, mainly in 3rd trimester of pregnancy.- As such aortic dissection is more common in males than females.- MC site of dissection is right lateral wall of ascending aorta, followed by descending aorta just below ligmentum venosum.

Correct Answer. b

Copyright © 2014 Delhi Academy of Medical Sciences, All Rights Reserved. 59/86

(132). Which of the following has the weakest association with Hepatocellular Carcinoma (HCC)

a. Throrotrast

b. Hepatitis C

c. Oral contraceptives

d. Smoking

Solution. (c) Oral contraceptivesReference – Read the text belowSol:Hepatitis B and Hepatitis C have known association with cirrhosis and Hepatocellular Carcinoma. Other significant risk factors are- Chronic alcohol abuse- Smoking- Aflatoxins- Nitrates, NItrites- Haemochromatosis, Wilson, Aplpha 1 Antitrypsin deficiemcy- Throrotrast- Associations with hormonal manipulations such as the use of oral contraceptive agents and anabolic steroids have been suggested butare weak

Correct Answer. c

(133). Which is the most common cause of Hemobilia

a. Blunt Trauma Abdomen

b. Iatrogenic injury

c. Cholelithiasis

d. Hepatic artery aneyrysm

Solution. (b) Iatrogenic injuryReference – Read the text belowSol:- Earlier Blunt Abdominal trauma was the most common cause of Hemobilia.- But recently with the advent of more interventional procedures like PTBD (Percutaneous Transhepatic Biliary Drainage), hemobilia hasbecome more common.

Correct Answer. b

(134). A biopsy of a posterior cervical lymph node in a 45-year-old nonsmoking woman reveals a poorly differentiated carcinoma. A chestradiograph, indirect laryngoscopy, and oral and thyroid examination are normal. Which of the following is the most appropriate next stepin the management of this patient?

a. A sputum cytology

b. A blind biopsy of the nasopharynx

c. An iodine-131 (131 I) scan of the thyroid

d. Bronchoscopy

Solution. (b) A blind biopsy of the nasopharynxReference – Read the text belowSol:- Whenever a cervical lymph node contains a poorly differentiated carcinoma with no obvious primary site, a blind or direct visualizationbiopsy should be taken of the nasopharynx to rule out a nasopharyngeal carcinoma.- These tumours have a putative relationship with Epstein-Barr virus, and are very common in china.- Recent development of an Epstein-Barr virus vaccine in Britain may eradicate this disease.

Correct Answer. b

Copyright © 2014 Delhi Academy of Medical Sciences, All Rights Reserved. 60/86

(135). Deep vein thrombosis occurs most commonly after:

a. Total hip replacement

b. Gastrectomy

c. Prostatic Operation

d. Brain surgery

Solution. (a) Total hip replacementRef.: Bailey - 936Sol :- Deep vein thrombosis can occur after any major surgery, leading to prolonged immobilization.Management of DVT- Prophylaxis : As most conditions associated with high risk of DVT are known prophylaxis is best way to avoid DVT.Methods of prophylaxis include :- Low molecular weight heparin (LMWH)- Low dose heparin- Elastic stockings (ES)- Intermittent pneumatic compression- Warfarin- Aspirin therapy alone is not sufficient for DVTTreatment :- Patient who are confirmed to have DVT should be started LMWH and rapidly anticoagulated with warfarin.- In iliofermoral DVT surgical thrombolysis should be considered in patient with impending venous gangrene and phlegmasia ceruleandolens.- In a case of pulmonary embolism secondary to DVT thrombolytic therapy should be initiated earliest, agents include : Recombinanttissue plasminogen activator (rtPA), urokinase, streptokinase.

Correct Answer. a

(136). In primary hyperparathyroidism, which one of the following is the most common finding?

a. Solitary adenoma

b. Double adenoma

c. Hyperplasia of two or more glands

d. Carcinoma

Solution. (a) Solitary adenomaRef.: Harrison’s Principles of Internal Medicine, (17th Ed.,) Pg. 2380Sol :- Primary hyperparathyroidism result from enlargement of single gland or parathyroid adenoma in approximately 80% of cases multipleadenoma or hyperplasia in 15 – 20% of cases and parathyroid carcinoma in 1% cases.- Harrison writes, a single abnormal gland is the cause in 80% of cases.

Correct Answer. a

(137). Most common bacterial infection in lymph edema is?

a. Staphylococcus

b. Streptococcus

c. E. Coli

d. Pseudomonas

Solution. (b) StreptococcusRef.: Read the text belowSol :- Tissues with lymphedema are at risk of infection. Cellulitis is both a cause and complication of lymphedema.- The most common complication of both primary and secondary lymphedema is erysipelas (acute streptococcus bacterial infection of thedeep epidermis with lymphatic spread).- Cellulitis may occur concurrently in lymphedema decause the pooling of protein-rich lymph fluid makes it easier for the patient todevelop an infection.

Correct Answer. b

Copyright © 2014 Delhi Academy of Medical Sciences, All Rights Reserved. 61/86

Copyright © 2014 Delhi Academy of Medical Sciences, All Rights Reserved. 62/86

(138). A 26-year-old woman presents with a palpable thyroid nodule shown in the image and needle biopsy demonstrates amyloid in palpablelymph nodes on the same side as the lesion. The preferred treatment should be:

a. Removal of the involved node, the isthmus, and the enlarged lymph node.

b. Removal of the involved lobe, the isthmus, a portion of the opposite lobe, and the enlarged lymph node

c. Total thyroidectomy and modified neck dissection on the side of the enlarged lymph node.

d. Total thyroidectomy and irradiation of the cervical lymph nodes.

Solution. (c) Total thyroidectomy and modified neck dissection on the side of the enlarged lymph node.Ref: Read the text belowSol: Presence of thyroid nodule along with palpable cervical lymph nodes and amyloid histologically is clearly indicative of Medullaryvariety of Carcinoma

Correct Answer. c

Copyright © 2014 Delhi Academy of Medical Sciences, All Rights Reserved. 63/86

(139). Parathyroid adenoma most commonly involves which of the following sites:-

a. Thyroid substance

b. Superior parathyroid lobe

c. Inferior parathyroid lobe

d. In the mediastinum

Solution. (c) Inferior parathyroid lobeRef: Read the text belowSol:Sites of parathyroid adenoma (in decreasing order)- Inferior parathyroid gland- Thymus- Thyroid- Pericardium- Behind the oesophagus

Correct Answer. c

(140). All of the following are correct statements about radiological evaluation of a patient with Cushing’s syndrome except:-

a. MRI of the sella turcica will identify a pituitary cause for Cushing’s syndrome

b. Petrosal sinus sampling is the best way to distinguish a pituitary tumor from an ectopic ACTH producing carcinoma

c. Adrenal CT scan distinguishes adrenal cortical hyperplasia from an adrenal cortical hyperplasia from an adrenal tumor.

d. MRI of adrenal may distinguish adrenal adenoma from adrenal tumor.

Solution. (a) MRI of the sella turcica will identify a pituitary cause for Cushing’s syndromeRef: Read the text belowSol:- MRI of the sella turcica will identify a pituitary cause for Cushing’s syndrome- Cushing disease is caused by microadenoma of pituitary and both CT scan and MRI scan cannot reliably detect these tumors.- Petrosal sinus sampling is the best way to distinguish a pituitary tumor from an ectopic ACTH producing carcinoma- Adrenal CT scan distinguishes adrenal cortical hyperplasia from an adrenal cortical hyperplasia from an adrenal tumor.- MRI of adrenal may distinguish adrenal adenoma from adrenal tumor.

Correct Answer. a

(141). The thyroiditis also known as “Painless Thyroiditis”-

a. Subacute lymphocytic thyroiditis

b. Dequervain’s

c. Hashimoto

d. Riedel

Solution. (a) Subacute lymphocytic thyroiditisRef: Read the text belowSol:- “Painless Thyroiditis” or silent thyroiditis occurs in patients with underlying autoimmune thyroid disease.- Clinical course is similar to that of subacute thyroiditis except that there is little or no thyroid tenderness.- The condition occurs in 5% of women 3-6 months after pregnancy and is termed postpartum thyroiditis.

Correct Answer. a

Copyright © 2014 Delhi Academy of Medical Sciences, All Rights Reserved. 64/86

(142). All are true about paget disease except:

a. 97% associated with underlying invasive carcinoma of breast

b. 50% are hormone receptor positive

c. Wedge or punch biopsy is taken from nipple for diagnosis

d. Underlying tumor lying within 2 cm for the nipple

Solution. (b) 50% are hormone receptor positiveRef.: Read the text belowSol :- Nearly 98% of mammary paget disease cases are associated with an underlying carcinoma, either in situ (intraductal, 10%) orinfiltrating cancer (90%)- Scrape cytology has been suggested as a noninvasive and reliable, rapid diagnostic screening method for mammary paget disease.- Punch, wedge, or excisional biopsy of the lesional skin of the nipple areola complex to include the dermal and subcutaneous tissue fordetailed microscopic examination provides an adequate sample for the accurate diagnosis of mammary Paget disease.- In cases of positive estrogen and progesterone receptors in an underlying breast carcinoma, the overlying paget disease is negative forthese receptors.

Correct Answer. b

(143). During breast reconstruction surgery which of the following structure is preserved?

a. Pectoralis minor

b. Pectoralis major

c. Serratus anterior

d. Nipple areola complex

Solution. (b) Pectoralis majorRef.: Read the text belowSol :- “As the breast implant is mostly placed beneath the pectoralis major muscle, it is never dissescted.- A complete musculofasical pocket is created by lateral and inferolateral dissection of the pectoralis minor and serratus anterior.- If indicated, mastectomy is performed with preservation of skin envelop, but nipple areola complex is included in the resection withfuniform incision that is more oblique inferiorly.

Correct Answer. b

(144). Which one is not true regarding Buerger’s disease?

a. Men are usually involved

b. Occurs below 50 years of age

c. Smoking is predisposing factor

d. Veins and nerves are never involved.

Solution. (d) Veins and nerves are never involved.Ref.: Short case S. Das - 161Sol :- Buerger’s disease is the inflammatory reaction in the arterial wall with involvement of neighbouring veins and nerves, terminating inthrombosis of a vessel.

Correct Answer. d

Copyright © 2014 Delhi Academy of Medical Sciences, All Rights Reserved. 65/86

(145). When splenectomy is done for ITP , platelet infusion is given at :

a. 2 Hours before surgery

b. After incision

c. Immediately after ligating splenic vein

d. Immediately after removal of spleen

Solution. (c) Immediately after ligating splenic veinRef.: Schwartz 8/e, p 1306Sol :- “Patient with extreme low platelet count (<10000/mm3) should have platelet available during splenectomy but should not receivepreoperatively. Once the splenic pedicle is ligated, platelet are given to those who continue to bleed”

Correct Answer. c

(146). Splenectomy is not done in:

a. Myelofibrosis

b. Sickle cell anaemia

c. Hereditary sphercocytosis

d. Splenic abscess

Solution. (b) Sickle cell anaemiaRef.: CSDT 12/e 632Sol :- Splenectomy is absolutely indicated in cases of hereditary spherocytosis and splenic abscess- In myelofibrosis splenectomy may be necessary if splenomegaly impairs alimentations and should be performed before cachexia sets in- In sickle cell anaemia splenectomy is indicated very rarely if there is hypersplenism or acute sequestration crisis.

Correct Answer. b

(147). Hypersplenism is seen in all of the conditions except :

a. Pancytopenia

b. Thrombocytopenia

c. Leucopenia

d. Polycythemia

Solution. (d) PolycythemiaRef.: Bailey - 1104Sol :- Hypesplensim is a clinical syndrome characterized by splenic enlargement, any combination of anaemia leucopenia orthrombocytopenia with compensatory bone marrow hyperplasia and improvement after splenectomy.

Correct Answer. d

Copyright © 2014 Delhi Academy of Medical Sciences, All Rights Reserved. 66/86

(148). Which of the following is the most common presenting symptom of non-cirrhotic portal fibrosis

a. Chronic liver disease

b. Ascites

c. Upper GI bleed

d. Encephalopathy

Solution. (c) Upper GI bleedRef.: Read the text below Sol :- Non-cirrhotic portal hypertension (NCPH) comprises diseases having an increase in portal pressure (PP) due to intraheptic orprehepatic lesions, in the absence of cirrhosis. The lesions are generally vascular, either in the portal vein, its branches or in theperisinusoidal area.- Because the wedged hepatic venous pressure is near normal, measurement of intravariceal or intrasplenic pressure is needed to assessPP. The majority of diseases included in the category of NCPH are well-characterized disease entities where portal hypertension (PHT) isa late manifestation and, hence, these are not discussed.- Two diseases that present only with features of PHT and are common in developing countries are non-cirrhotic portal fibrosis (NCPF)and extrahepatic portal vein obstruction (EHPVO). Non-cirrhotic portal fibrosis is a syndrome of obscure etiology, characterized by'obliterative portovenopathy' leading to PHT, massive splenomegaly and well-tolerated episodes of variceal bleeding in young adults fromlow socioeconomic backgrounds, having near normal hepatic functions.- In some parts of the world, NCPF is called idiopathic portal hypertension (IPH) or 'hepatoportal sclerosis'. Because 85-95% of patientswith NCPF and EHPVO present with variceal bleeding, treatment involves management with endoscopic sclerotherapy (EST) or varicealligation (EVL).

Correct Answer. c

(149). Indications of surgical drainage in amoebic liver abscess are all except:

a. Left lobe abscess

b. Deep and less than 5 mm in size

c. Multiple

d. Septicemia from secondary infection.

Solution. (b) Deep and less than 5 mm in sizeRef.: Maingot 11/e, p 762Sol :Surgical drainage-Indications•- Left lobe abscess not amenable to percutaneousdrainage•- Life threatening haemorrhage with or withoutintraperitoneal rupture of abscess. Amoebic abscess eroding into neighbouringstructures•- Septicemia from secondary infection• -Failure of response to conservative therapy

Correct Answer. b

Copyright © 2014 Delhi Academy of Medical Sciences, All Rights Reserved. 67/86

(150). Solitary hypoechoic lesion of the liver without septae or debris is most likely to be:

a. Hydatid cyst

b. Caroli’s disease

c. Simple cyst

d. Malignancy

Solution. (c) Simple cystRef.:Rumack’s Ultrasound 3/e 86Sol :- Solitary hypoechoic lesion without septa or debris is most likely to be a single cyst.

Correct Answer. c

(151). For open pneumothorax which of the following is management of choice?

a. IPPV

b. ICD with underwater seal

c. Thoracostomy and close the rent

d. Wait and watch

Solution. (b) ICD with underwater sealRef.: CSDT - 349Sol : - Immediate management is to cover the wound with a rectangular sterile occlusive dressing that is closed securely with tape ononly 3 sides. -Thus, the dressing prevents atmospheric air from entering the chest wall during inspiration but allows any intrapleural air out duringexpiration. -Tube thoracostomy should be done when the patient is stabilized. -The wound may require later surgical repair.

Correct Answer. b

Copyright © 2014 Delhi Academy of Medical Sciences, All Rights Reserved. 68/86

(152). Which is not true regarding thoracic inlet syndrome:

a. Most commonly radial nerve is involved

b. Resection of 1st rib is effective t/t

c. Physiotherapy and position exercises relieves symptom

d. Neurological signs and symptoms are common

Solution. (a) Most commonly radial nerve is involvedRef.: CSDT 12/e 1821Sol :- In thoracic outlet syndrome lower trunk of brachial plexus (Ulnar nerve division is compressed

Correct Answer. a

(153). Muscle not cut in posterolateral thoracotomy is :

a. Serratus anterior

b. Lattisimus dorsi

c. Intercostal muscles

d. Pectoralis major

Solution. (d) Pectoralis majorRef.: Read the text belowSol :

Correct Answer. d

Copyright © 2014 Delhi Academy of Medical Sciences, All Rights Reserved. 69/86

(154). Secondary deposits from carcinoma breast are commonest in?

a. Spleen

b. Liver

c. Brain

d. Bone

Solution. (d) BoneRef.: Read the text belowSol :- “The commonest malignancies that metastasise to the spine include breast (21%), lung (14%) prostate (7.5%), renal (5%),gastrointestinal (5%) and thyroid (2.5%).- The most frequent pathway for spread of metastatic deposits is through the venous system”- Metastasis in order of requency are lumbar vertebrae, femur, thoracic vertebrae, rib and skull are affected and these deposits aregenerally osteolytic.- Metastases may also commonly occur in the liver, lungs and brain and, occasionally, the adrenal glands and ovaries.- Batson’s vertebral venous plexus, which invests the vertebrae and extends from the base of the skull to the sacrum, may provide a routefor breast cancer metastases to the vertebrae, skull, pelvic bones, and central nervous system.

Correct Answer. d

(155). A30-year-old man is noted to be anemic, with clinical jaundice and a palpable spleen on abdominal exam. Splenectomy is the onlytreatment for this patient’s autosomal dominant disorder. Select the most likely diagnosis.

a. Thalassemia

b. Hereditary spherocytosis

c. Sickle cell disease

d. Thrombotic thrombocytopenic purpura (TPP)

Solution. (b) Hereditary spherocytosisRef: Read the text belowSol :- Hereditary spherocytosis is the most common symptomatic familial hemolytic anemia, and is transmitted as an autosomal dominanttrait.- A defect in the red cell membrane causes increased trapping in the spleen and hemolysis. Anemia, jaundice, and splenomegaly areclinical findings. Splenectomy is the only treatment.- Thalassemia is transmitted as a dominant trait; anemia is the result of a defect in hemoglobin synthesis.- Thalassemia major, or homozygous thalassemia, is associated with anemia, icterus, splenomegaly, and early death.- Transfusions are usually required. Splenectomy may reduce hemoloysis and transfusion requirements. Sickle cell anemia is hereditaryhemolytic anemia.- Serum bilirubin may be mildly elevated.- Splenomegaly often precedes autoinfraction. Splenectomy may be indicated for chronic hypersplenism or acute splenic sequestration

Correct Answer. b

(156). All are features of Fournier’s Gangrene except :

a. Testicles are involved

b. Obliterative arteritis seen

c. Hemolytic streptococci, isolated

d. Necrotizing fasciitis

Solution. (a) Testicles are involvedRef.: Read the text belowSol :- Fournier gangrene is a necrotizing infection that involves the soft tissues of the male genitalia.- Fournier gangrene is a specific form of necrotizing fasciitis.- The corpora cavernosa, urethra, testes and cord structures are usually spared in Fournier gangrene, while the superficial and deepfascia and the skin are destroyed.

Correct Answer. a

Copyright © 2014 Delhi Academy of Medical Sciences, All Rights Reserved. 70/86

(157). Most common tumor of anterior mediastinum is :

a. Thymoma

b. Lymphoma

c. Germ cell tumor

d. Thyroid tumors

Solution. (a) ThymomaRef.: Read the text belowSol :- Anterior mediastinal compartment, also known as anterosuperior compartment, is anterior to the pericardium.- Masses in this area is more likely to be malignant than those in other compartments.- Most common masses found here include thymomas, germ cell tumors, lymphomas, thyroid tissue, and parathyroid lesion.

Correct Answer. a

(158). After rupture of middle meningeal artery bleeding occurs in which region :

a. Subdural bleed

b. Epidural bleed

c. Intracerebral bleed

d. Subarachnoid bleed

Solution. (b) Epidural bleedRef.: Bailey - 303Sol :- An injured middle meningeal artery is the cause of an epidural hematoma. A head injury (e.g. from a road traffic accident or sportsinjury) is required to rupture the artery. Emergency treatment requires decompression of the haematoma, usually by craniotomy.Subdural bleeding is usually venous in nature, rather than arterial.- The middle meningeal artery runs in a groove on the inside of the cranium.

Correct Answer. b

(159). Which of the following is most appropriately termed as penetrating neck injury ?

a. 2cm deep

b. Injury of vital structure

c. Breach of platysma

d. Through and through wound

Solution. (c) Breach of platysmaRef.: Subiston - 139Sol :- Injuries that do not penetrate the platysma can be considred superficial and no further investigation is needed.- Wounds that penetrate the platysma must be further evaluated.

Correct Answer. c

Copyright © 2014 Delhi Academy of Medical Sciences, All Rights Reserved. 71/86

(160). A 40-year-old male brough to the emergency room with a stab injury to the chest. On examination patient is found to be hemody-namically stable. The neck veins are engorged and the heart sound are muffled. The following statements are true for this patient except:

a. Cardiac tamponade is likely to be present

b. Immediate emergency room thoracotomy to be done

c. Echocardiogram should be done to confirm pericardial blood

d. The entry wound should be sealed with an occlusive dressing

Solution. (b) Immediate emergency room thoracotomy to be doneRef.: Read the text belowSol :Treatment :- Treatment is of traumatic cardiac tamponade is thoracotomy, pericardiocentesis has no role in the management of such tamponade. ….- In hemodynamic ally unstable patient emergency room thoracotomy should be performed so as to save the life.- In this case too operative room thoracotomy (not emergency room thoracotomy) is required.- Emergerncy room thoracotomy means thoracotomy in emergency room without proer planning, it is indicated only in grave situations.

Correct Answer. b

Copyright © 2014 Delhi Academy of Medical Sciences, All Rights Reserved. 72/86

(161). The most common type of trachea-oesophageal fistula is :

a. Esophageal atresia without tracheoesophageal fistula

b. Esophageal atresia with proximal tracheoesophageal fistula

c. Esophageal atresia with distal tracheoesophageal fistula

d. Esophageal atresia with proximal and distal fistula

Solution. (c) Esophageal atresia with distal tracheoesophageal fistula Ref.: Read the text below Sol : Fistulae between the trachea and esophagusin the newborn can be of diverse morphology and anatomical location; however, various pediatric surgical publications have attempted aclassification system based on the below specified types. Not all types include both esophageal agenesis and tracheoesophageal fistula, but themost common types do.

Correct Answer. c

Copyright © 2014 Delhi Academy of Medical Sciences, All Rights Reserved. 73/86

(162). Umbilical hernia in a child – indication for surgery

a. Failure to disappear by 3 years

b. > 2 cm size

c. Symptomatic

d. All of the above

Solution. (d) All of the aboveRef.: Read the text belowSol :- When the orifice is small (< 1 or 2), 90% closes within 3 years, and if these hernias are asymptomatic, reducible, and don’t enlarge, nosurgery is needed (and in other cases it must be considered).

Correct Answer. d

(163). In performing hepatic resection, a knowledge of the different lobes and segments of the liver is mandatory. The right and left lobes of theliver are separated by an imaginary plane (Cantlie’s line) that passes between the the inferior vena cava (IVC) and which of thefollowing?

a. Portal vein

b. Falciform ligament

c. Left margin of the quadrate lobe

d. Gallbladder

Solution. (d) GallbladderRef: Read the text belowSol :- The hepatic artery, portal vein, and hepatic bile duct are distributed equally between both lobes of the liver divided by Cantlie’s line.- This line passes between the inferior vena cava posteriorly and the gallbladder fossa anteroinferiorly.- The falciform ligament does not divide the liver into a right and left lobe; it divides the true left lobe into medial and lateral segments.- The caudate and quadrate lobes are part of the left lobe, and, thus, Cantlie’s line passes along their right (and not left) margins.

Correct Answer. d

(164). A 65-year-old female on her routine examination was noted to have a pulsatile abdominal mass. She has been otherwise healthy withhistory of hypertension with no other history, except family history of father dying of ruptured abdominal aortic aneurysm. What are theacceptable reasons to operate on abdominal aortic aneurysms in 65-year-old female with 5-cm infrarenal aneurysm?

a. Presence of aneurysm

b. Aneurysm with intramural thrombus

c. Asymptomatic aneurysm 5.5 cm

d. Associated 2-cm iliac aneurysm

Solution. (c) Asymptomatic aneurysm 5.5 cmRef: Read the text belowSol :- The current indication for repair of abdominal aortic aneurysm in female includes aneurysm size 5 cm in acceptable risk patient.- A small aneurysm study has increased the size that could be observed to 5.5 cm in male while in female it is acceptable to treataneurysm at 5 cm size for acceptable risk. Any aneurysm with associated complication should be treated; just the presence of intramuralthrombus does not justify repair.- Asymptomatic 5.5-cm aneurysm should be treated in all patients, male or female, at acceptable cardiac risk.- Patients with 2-cm aneurysm of iliac artery without any symptoms and complications should be observed; as the risk of surgery ishigher than risk of observation till they reach to 4 cm.- In patients, not in child-bearing age, 1.5-cm splenic aneurysm could be observed

Correct Answer. c

Copyright © 2014 Delhi Academy of Medical Sciences, All Rights Reserved. 74/86

(165). Thyrotoxic myopathy affects

a. Muscles of the neck

b. Proximal limb muscles

c. Distal limb muscles

d. Muscles of the tongue

Solution. (b) Proximal limb musclesRef: Harrison’s- 2693.Sol:- Thyroid hormone deficiency states result in neurological syndromes that vary depending on the age of onset of the deficiency. Muscleweakness occurs most prominently in the adult forms of myxedema.- Thyroid hormone excess also results in Proximal myopathy. Thyrotoxic myopathy is believed to be secondary to a disturbance in thefunction of the muscle fibers from increased mitochondrial respiration, accelerated protein degradation and lipid oxidation, andenhanced beta-adrenergic sensitivity due to excessive amounts of thyroid hormone.- Thyroid disorders may result in orbital myositis, a disorder than may impair ocular movement and therefore appear clinically as eyemuscle weakness

Correct Answer. b

(166). A60-year-old male patient with bilateral carotid artery stenosis 90%, with history of right-sided weakness with resolution of symptoms in15 minutes. How would you treat the patient?

a. Right carotid endarterectomy

b. Left carotid endarterectomy

c. Right carotid angioplasty and stenting

d. Start patient on aspirin

Solution. (b) Left carotid endarterectomyRef: Read the text belowSol :- The treatment for symptomatic carotid artery stenosis greater than 70% is carotid endarterectomy. Since patient has left cerebralsymptoms, it would be appropriate to treat that side first.- Patient would need bilateral carotid endarterectomy but symptomatic side would be the first one to be operated.- Heparin has no significant role in preventing stroke. Aspirin isa part of therapy but would not constitute a primary modality fortreatment.

Correct Answer. b

(167). Which of the following tumor cell types has the worst prognosis in the thyroid gland ?

a. Lymphoma

b. Anaplastic carcinoma

c. Medullary carcinoma

d. Papillary carcinoma

Solution. (b) Anaplastic carcinomaRef: Read the text belowSol :- Anaplastic carcinoma is essentially an untreatable disease, with nearly uniformly fatal outcome despite the treatments that have beentried. It spreads quickly and early and is not controlled by radioiodine, radiation or chemotherapy.- Extensive surgical efforts have resulted in some relief of obstructive disease while tumor spread usually continues unchecked. In orderof malignant behavior, medullary thyroid carcinoma would be next worst, followed by follicular, lymphoma (which may present primarilyin the thyroid gland) and papillary with the best prognosis.

Correct Answer. b

Copyright © 2014 Delhi Academy of Medical Sciences, All Rights Reserved. 75/86

(168). Total thyroidectomy is an operation frequently carried out, but total parathyroidectomy should be avoided at some considerable effort.The principle reason for this is

a. Total parathyroidectomy has a much higher risk to the recurrent laryngeal nerves

b. There is no satisfactory replacement for endogenous parathormone

c. Ablation of the parathyroid glands endangers the blood supply to the anterior cervical spinal cord

d. Parathyroidectomy is to technically difficult, and can only be done with an operating microscope

Solution. (b) There is no satisfactory replacement for endogenous parathormone Ref: Read the text belowSol :- A patient who is aparathyroid is suffering a crippling disorder with continuous requirement of vitamin D and calcium to inadequatelymaintain a serum calcium always dangerously close to tetany levels. The other technical and biologic reasons listed are trivial for thetechnical reasons and false in the biologic assertions.- It is true that thyroid cancer is more prevalent than parathyroid cancer, but that would not means total ablation of all parathyroidglands, since it is usually a unifocal disease, and ablation of the other parathyroids does not enhance the uptake of any given treatmentdirected at the parathyroid.- Total parathyroidectomy or its functional equivalent by interrupting its blood supply is altogether too easy, and is sometimesunintentionally preservation procedure”.

Correct Answer. b

(169). Klatskin tumor involves

a. Intrahepatic bile duct

b. Hepatic duct confluence

c. Lowest 1/3 bile duct

d. Periampullary area

Solution. (b) Hepatic duct confluenceRef: Read the text belowSol:- Nodular tumors arising from the part of the common bile duct between the cystic duct junction and the confluence of the right and lefthepatic ducts at the liver hilus are called Klatskin tumors.- They are often associated with a collapsed gallbladder, a finding that mandates visualization of the entire billary tree.- Klatskin tumors are slow growing, have marked sclerosing characteristics and distal metastases are infrequent.

Correct Answer. b

(170). The commonest cause of death in patients with alcoholic cirrhosis following portosystemic shunting is

a. Bleeding esophageal varices

b. Hepatic failure with encephalopathy

c. Malnutrition

d. Helpatocellular carcinoma

Solution. (b) Hepatic failure with encephalopathyRef: Davidson’s-961;Sol:- The most common cause of death after transjugular intrahepatic portosystemic shunting is hepatic failure.- Although portosystemic shunt surgery prevents recurrent bleeding, it carries a high mortality and often leads to encephalopathy.- Survival is not prolonged and death from liver failure occurs.- In practice, portosystemic shunts are now reserved for patients in whom other treatments have not been successful and are offered onlyto those with good liver function.

Correct Answer. b

Copyright © 2014 Delhi Academy of Medical Sciences, All Rights Reserved. 76/86

(171). Initial investigation for an amoebic liver abscess is

a. USG

b. Exploratory laparotomy

c. Ct scan

d. Technetium-99 scan

Solution. (a) USGRef: Internet resourcesSol :I. Ultrasonography is the preferable initial diagnostic test.It is rapid, inexpensive, and is only slightly less sensitive than CT scan (75-80% sensitivity vs 88-95% for CT scan). Ultrasonographysimultaneously evaluates the gallbladder and avoids radiation exposure.- As opposed to scanning with technetium-99m, sonography often can distinguish an abscess from a tumor or other solid focal lesion.- The lesions tend to be round or oval, with well-defined margins, and hypoechoic.

Correct Answer. a

(172). Classic diagnostic point for an uncomplicated indirect inguinal hernia is

a. Impulse on coughing

b. Positive deep ring occlusion test

c. Groin swelling

d. Reducibility

Solution. (b) Positive deep ring occlusion testRef: Das textbook of surgery, Pg 651.Sol:- Impulse on coughing, groin swelling and reducibility are features of both direct and indirect inguinal hernias.- The classic diagnostic point to differentiate between the two is a positive deep ring occlusion test.Deep ring occlusion test; Patient is asked to lie down and the hernia is reduced. The position of deep inguinal ring is marked out.- The deep inguinal ring lies 1.25 cm above the mid inguinal point which is situated at the midpoint between anterior superior iliac spineand symphisis pubis. The thumb is placed over the deep ring and patient is asked to cough.- Look whether any cough impulse is seen medial to the deep ring. If no expansile impulse is seen in lying down position, patient is askedto stand with the deep ring occluded and is asked to cough again.

Correct Answer. b

(173). Which of the following statements about lymphatic capillaries are true?

a. These vessels have delicate tricuspid valves every 2 to 3 mm.

b. Lymphatic capillaries are more permeable than blood capillaries.

c. Lymphatic capillaries are less permeable than blood capillaries.

d. Lymphatic capillaries contain gaps large enough to admit particles as large as lymphocytes.

Solution. (d) Lymphatic capillaries contain gaps large enough to admit particles as large as lymphocytes.Ref - Read the text belowSol:- The transporting lymphatic vessels have valves but lymphatic capillaries do not.- The lymphatic capillaries will accept particles including bacteria, red blood cells, and lymphocytes and transport them to regionallymph nodes.

Correct Answer. d

Copyright © 2014 Delhi Academy of Medical Sciences, All Rights Reserved. 77/86

(174). Vicryl, the commonly used suture material is a :

a. Homopolymer of polydiaxone

b. Co-polymer of glycolide and lactide

c. Homopolymer of glycolide

d. Homopolymer of lactide

Solution. (b) Co-polymer of glycolide and lactideRef: Bailey and Love- 238Sol:- Vicryl is polyglactin is a Synthetic, Absorbable, Multifilament, lubricant coated suture.- It is a copolymer of lactide and glycolide in a 90:10 ratio, coated with plyglactin and calcium stearate.- Complete absorption by 60-90 days; (increases in alkaline environment and high temps; don’t use to close bladder).

Correct Answer. b

(175). Commonest cause of A-V fistulae is?

a. Congenital

b. Traumatic

c. Iatrogenic

d. Tumour erosion

Solution. (c) IatrogenicRef.: Read the text belowSol :- An arteriovenous fistula is an abnormal communication betweenthe arterial and the venous circulations that bypasses the capillarybed.- The most common type is an acquired fistula (65-75% of cases)- The most common cause of arteriovenous fistulas is iatrogenic

Correct Answer. c

(176). Which of the following conditions is associated with increased risk of breast cancer?

a. Fibrocystic mastopathy.

b. Severe hyperplasia.

c. Atypical hyperplasia.

d. Papillomatosis.

Solution. (c) Atypical hyperplasiaReferences: Read the text below.Sol:- Fibrocystic mastopathy, or fibrocystic disease, was once thought to increase the risk of breast cancer; however, later studies of thepathologic findings in fibrocystic complex found an increased cancer risk only for patients whose biopsies showed atypical hyperplasia.- “Severe hyperplasia” is a pathologic term that refers to the amount of hyperplasia and is frequently seen in the biopsy specimens ofyoung women; it is a misleading term and is not associated with a disease risk.- Papillomatosis is also part of the fibrocystic complex and is a frequent finding in benign breast biopsies; it does not confer an increasedrisk of cancer.

Correct Answer. c

Copyright © 2014 Delhi Academy of Medical Sciences, All Rights Reserved. 78/86

(177). The following comments about immune thrombocytopenic purpura (ITP) are accurate:

a. Platelet count is low.

b. Circulating antiplatelet factor is present.

c. Antiplatelet factor is immunoglobulin G (IgG) antibody.

d. Purpura is directed against a platelet-associated antigen.

Solution. (a) Platelet count is low.References: Read the text below.Sol:- ITP is a disease characterized by low platelet counts and a bone marrow with proliferative megakaryocytes- The disease is usually diagnosed by abnormal episodes of bleeding, which may occur from trauma, menses, etc. The causative event bywhich sensitized platelet-associated antibodies are developed is probably multiple.- The effect of the platelet-associated antibodies, which is usually IgG, is directed against a platelet-associated antigen and, when coatingof normal platelets occurs, results in the platelets being sequestered from the system by the reticuloendothelial system with resultantthrombocytopenia.

Correct Answer. a

(178). A 57-year-old male patient with a 60 pack-year smoking history is referred for a 1.5-cm solitary mass in the right upper lobe. CT scandemonstrates no evidence of lymph node involvement. What should further workup or treatment include?

a. Radiation therapy

b. Open lung biopsy

c. Chemotherapy

d. Right upper lobectomy

Solution. (d) Right upper lobectomy.References: Read the text below.Sol:- The appropriate treatment is surgical lobectomy. Observation with repeat chest x-ray is not warranted with a smoking history.- This patient is in clinical stage I, based on tumor size and nodal status. There is no clear benefit in biopsying the lesion.- Chemotherapy and radiation may be indicated in certain stage IIIa lesions or in locally advanced disease.

Correct Answer. d

(179). A 22-year-old female is referred for evaluation of a 2-cm posterior mediastinal mass discovered on routine chest radiograph. What is themost likely diagnosis?

a. Bronchogenic cyst

b. Lymphoma

c. Neurogenic tumor

d. Thymoma

Solution. (c) Neurogenic tumorReferences: Read the text below.Sol:- The most common posterior mediastinal mass is a neurogenic tumor. Seventy-five percent of neurogenic tumors occur in children under4 years of age.- Childhood tumors are more likely to be malignant. Lymphoma, thymoma, and germ cell tumors are commonly located in the anteriormediastimun.- Middle mediastinal lesions include bronchogenic and pericardial cysts. Metastatic adenocarcinoma may involve the pleural surfaces;however, lesions are often small and multiple.

Correct Answer. c

Copyright © 2014 Delhi Academy of Medical Sciences, All Rights Reserved. 79/86

(180). Harvey’s sign is?

a. Transmitted pressure wave on coughing in a varicose vein

b. Related to the use of venous filling after emptying a length of vein

c. Loss of hair from eyebrows

d. None of the above

Solution. (b) Related to the use of venous filling after emptying a length of veinRef.: Read the text belowSol :- Delayed venous refilling is called Harvey’s sign, signifies ischemia. Venous refilling is increased in AV fistula.

Correct Answer. b

(181). A patient presents in coma for 20 days, what will be the best way to give him nutrition?

a. Parenteral nutrition

b. Ryle’s tube feeding

c. Feeding via jejunostomy

d. Oral feeding

Solution. (c) Feeding via jejunostomyRef.: Read the text belowSol :- Oral feeding is rarely applicable to comatose patients. Early enteral (EN) nutrition through a nasoduodenal nasojejunal tube willovercome the problem of delayed gastric emptying. Patients requiring enteral feeding on a long-term basis should have a gastrostomy orgastrojejunostomy tube placed.- Parenteral nutrition (PN) is indicated when a patient’s gastrointestinal tract does not tolerate full enteral feeding or when the access tothe GI tract is difficult after head and neck trauma.- Parenteral nutrition may be administered as central venous nutrition (expected duration of PN > 7 days) or as peripheral venousnutrition (short-term PN< 7 days

Correct Answer. c

(182). The most typical presentation of the hypercalcemic patients proven by operation to have primary hyperparathyroidism currently is

a. Asymptomatic

b. Renal stones

c. Bone pain and radiographic evidence of resorption

d. Abdominal pain

Solution. (a) AsymptomaticRef:Read the text belowSol:- Hyperparathyroidism is now nearly uniformly found as an incidental biochemical abnormality on screening blood examination.- It is probably the most valuable yield of the multichannel autoanalyzer, since hypercalcemia is always worth investigation for discoveryof diseases that are either treatable, significant with respect to prognosis, or when correction would prevent further problems.- The old mnemonic “stones, bones, moans, and abdominal groans” defines the past pattern of advance stage disease which broughtpatients to have these complaints investigated.- Retrospectively, there is some insidious loss of capability (forgetfulness, headache, many features attributed to advancing age) that areonly recognized by their absence when hyperparathyroidism is relieved.

Correct Answer. a

Copyright © 2014 Delhi Academy of Medical Sciences, All Rights Reserved. 80/86

(183). Each of the following constitutes a risk factor for breast cancer except :

a. Heredity

b. Fibrocystic disease

c. Nulliparity

d. Prolonged breast feeding

Solution. (d) Prolonged breast feedingRef:Read the text belowSol:- Earlier prior breast cancer in the patient or first order female relatives increases the risk of breast cancer as does prolonged estrogenstimulation as seen with nulliparity.- Dysplastic changes in fibrocystic disease may be precursors of neoplasia.- Prolonged breast feeding interrupts estrogen stimulation and is actually protective against the development of breast cancer.

Correct Answer. d

(184). In a 36 year old woman with a painless hard dominant breast lump, the next step should be

a. Mammography of the lump

b. Fine needle aspiration cytology

c. Radiation therapy

d. Radical mastectomy

Solution. (b) Fine needle aspiration cytologyRef:Read the text belowSol:- Mammography is not indicated for a mass that requires cytologic or histologic identification, so mammography of a suspicious lump willnot change the indication for its biopsy.- Neither radiotherapy nor mastectomy are appropriate until diagnositic confirmation of carcinoma is confirmed either by cytology orsubsequent biopsy at which point therapeutic options can be discussed, and bone scan would only be appropriate as a prediction ofmetastatic disease which still would require primary breast cancer diagnosis and treatment.

Correct Answer. b

(185). Skin grafts stored at 4°C can survive up to?

a. 1 week

b. 2 weeks

c. 3 weeks

d. 4 weeks

Solution. (b) 2 weeksRef.: Read the text belowSol :- Excess split-skin autografts harvested and meshed during a surgical session are often stored at short term for later burn surgery orgraft failure.- The current procedure in skin graft storage involves wrapping the meshed auto graft on a piece of ringer lactate or normal saline-moistened gauze, transferring it into a sterile container and storing it in a 4° C for 2 weeks.- The graft should never be totally immersed in saline because it will become macerated.- After 14 days of storage the respiratory activity of skin graft reduced by 50%

Correct Answer. b

Copyright © 2014 Delhi Academy of Medical Sciences, All Rights Reserved. 81/86

(186). A 40-year-old patient is suffering from carotid body tumor. Which of the following is the best choice of treatment for him:

a. Excision of tumor

b. Radiotherapy

c. Chemotherapy

d. Carotid artery ligation both proximal and distal to the tumor.

Solution. (a) Excision of tumorRef: Bailey - 732Sol : Treatment :- Surgical excision is only treatment available, as spontaneous regression is not reported.- As these tumor are highly vascular preoperative embolisation may be attempted so as to reduce vascularity.- In older individual with long history, the tumor may be left as such. In younger individuals if growth rate is high, tumor should beremoved.- Tumor is radio resistant.

Correct Answer. a

(187). True about branchial cyst

a. Seen deep to lower 1/3rd of sternocleidomastoid

b. Wall consist of lymphoid tissue

c. Lined by columnar epithelium

d. Present at birth.

Solution. (b) Wall consist of lymphoid tissueRef: Bailey - 770Sol :BRANCHIAL CYST- Develops from vestigial remnant of second branchial cleft.- Lined by squamous epithelium and contains thick turbid fluid full of cholesterol crystals.- Most common site – Upper part of neck at junction of upper third and middle third of sternocleidomastoid at its anterior border.Clinical features- Painless fluctuant swelling in upper and lateral part of neck in early or middle adulthood.- Swelling is soft in consistency with negative transillumination test.Complication- Recurrent infection due to presence of lymphoid tissue in its wall.- Acquired branchial fistulaDiagnosis – USG or fine needle aspiration Treatment – Complete excision

Correct Answer. b

(188). Bilateral pulseless disease in upper limbs is caused by :

a. Aortoarteritis

b. Coarctation of aorta

c. Fibromuscular dysplasia

d. Buerger’s disease

Solution. (a) AortoarteritisRef: Harrison - 1567Sol :- Bilateral pulseless disease in upper limbs is caused by aortoarteritis which is seen in Takayasu’s arteritis.

Correct Answer. a

Copyright © 2014 Delhi Academy of Medical Sciences, All Rights Reserved. 82/86

(189). The most important perforator of the lower limb is between

a. Long saphenous and posterior tibial vein

b. Short saphenous and posterior tibial vein

c. Short saphenous and popliteal vein

d. Long saphenous and femoral vein

Solution. (d) Long saphenous and femoral veinRef: Read the text belowSol :- The most important perforator of the lower limb is between is long saphenous and femoral vein.

Correct Answer. d

(190). Excision of hyoid bone is commonly done in :

a. Sublingual dermoid

b. Branchial fistula

c. Branchial cyst

d. Thyroglossal cyst

Solution. (d) Thyroglossal cystRef: Read the text belowSol :- Treatment for a thyroglossal cyst is surgical resection, often requiring concomitant removal of the midsection of the hyoid bone(Sistrunk procedure), to prevent recurrence.- Although generally benign the cyst will be removed if the patient exhibits difficulty in breathing or swallowing, or if the cyst is infected.Even if these symptoms are not present the cyst may be removed to eliminate the chance of infection or development of a carcinoma, orfor cosmetic reasons if there is unsightly protrusion from the neck.- The Sistrunk procedure involves excision not only of the cyst but also of the path's tract and branches. A removal of the central portionof the hyoid bone is indicated to ensure complete removal of the tract. It is unlikely that there will be a recurrence after such anoperation.- The original Sistrunk paper is available on-line with a modern commentary

Correct Answer. d

(191). Conservative surgery is not done in breast carcinoma if there is?

a. Lymph node metastasis

b. Sub-areolar lump

c. Lump of size 4 cms

d. Disease involving the lower quadrant only

Solution. (b) Sub-areolar lumpRef.: Read the text belowSol :Breast-conserving surgery is not suitable for all patients : it is not generally suitable for tumor- >5 cm (or for smaller tumors if the breast is small)- For tumors involving the nipple areola complex.- For tumors with extensive intraductal diseaseinvolving multiple quadrants of the breast.- For women with a history of collagen-vascular disease.- Do not have convenient access to radiation therapy.- Women who either do not have the motivation for breast conservation.

Correct Answer. b

Copyright © 2014 Delhi Academy of Medical Sciences, All Rights Reserved. 83/86

(192). All are true about basal cell carcinoma except:

a. Most common site is upper eye lid

b. Locally invasive

c. Rarely metastasizes

d. Associated with exposure to sun

Solution. (a) Most common site is upper eye lidRef.: Read the text belowSol :- Although most common cancer of the upper lip is basal cell carcinoma, most common site of basal cell carcinoma is not upper lip butnose”

Correct Answer. a

(193). Treatment of T4N0M0 stage of head and neck carcinoma is?

a. Surgery alone

b. Radiotherapy alone

c. Chemoradiation

d. Surgery and Radiotherapy

Solution. (d) Surgery and RadiotherapyRef.: Read the text belowSol :Treatment options for head and neck cancer- (T1 or T2 ) Early stages: surgery or radiation -(T3 or T4) Advanced stages:chemoradiation or surgery followed by radiation and chemotherapy- Very advanced cases: radiation and chemotherapy

Correct Answer. d

(194). Which of following is hypertonic?

a. 5%dextrose

b. 0.45% normal saline

c. 0.9% normal saline

d. 3% normal saline

Solution. (d) 3% normal salineRef.: Read the text belowSol :

Correct Answer. d

Copyright © 2014 Delhi Academy of Medical Sciences, All Rights Reserved. 84/86

(195). Lung injury with bad prognosis is?

a. Open pneumothorax

b. Closed pneumothorax

c. Tension pneumothorax

d. All have same prognosis

Solution. (c) Tension pneumothoraxRef.: Read the text belowSol :- As a general thing, the prognosis of open and closed pneumothorax is more favorable than that of tention pneumothorax because thelatter is apt to be atteneded with great tension of the gases, and marked isplacement of the adjacent structures.

Correct Answer. c

(196). Concept of carrel’s triangle is used for?

a. Urethral anastomosis

b. Vascular anastomosis

c. Bowel anastomosis

d. CBD anastomosis

Solution. (b) Vascular anastomosisRef: Read the text belowSol:

Correct Answer. b

(197). A patient presents with bilateral proptosis, heat intolerance and palpitations; most unlikely diagnosis here would be:

a. Hashimoto’s thyroiditis

b. Thyroid adenoma

c. Diffuse thyroid goiter

d. Reidel’s thyroiditis

Solution. (d) Reidel’s thyroiditisRef: Read the text belowSol:- The patient has features of hyperthyroidism which is not seen in Reidel’s thyroiditis.

Correct Answer. d

Copyright © 2014 Delhi Academy of Medical Sciences, All Rights Reserved. 85/86

(198). Which of the following is primarily a disease of nipple areola complex?

a. Duct papilloma

b. Paget’s disease

c. Periductal mastitis

d. Fibroadenoma

Solution. (b) Paget’s diseaseRef: Read the text belowSol :- Paget's (PAJ-its) disease of the breast is a rare form of breast cancer. Paget's disease of the breast starts on the nipple and extends tothe dark circle of skin (areola) around the nipple. Paget's disease of the breast isn't related to Paget's disease of the bone, a metabolicbone disease.- Paget's disease of the breast occurs most often in women older than age 50. Most women with Paget's disease of the breast haveunderlying ductal breast cancer, either in situ — meaning in its original place — or, less commonly, invasive breast cancer.- Only in rare cases is Paget's disease of the breast confined to the nipple itself.

Correct Answer. b

(199). Treatment of choice for flail Chest with respiratory failure is?

a. Only analgesia

b. Positive pressure ventilation

c. Open reduction plus internal fixation

d. Epidural anesthesia

Solution. (b) Positive pressure ventilationRef: Read the text belowSol :Treatment of the flail chest initially follows the principles of advanced trauma life support. Further treatment includes:- Good analgesia including intercostal blocks, avoiding narcotic analgesics as much as possible. This allows much better ventilation, withimproved tidal volume, and increased blood oxygenation.- Positive pressure ventilation, meticulously adjusting the ventilator settings to avoid pulmonary barotrauma.- Chest tubes as required.- Adjustment of position to make the patient most comfortable and provide relief of pain.- Aggressive pulmonary toiletSurgical fixation can help in significantly reducing the duration of ventilatory support and in conserving the pulmonary function.A patient may be intubated with a double lumen tracheal tube. In a double lumen endotracheal tube, each lumen may be connected to adifferent ventilator. Usually one side of the chest is affected more than the other, so each lung may require drastically different pressuresand flows to adequately ventilate.

Correct Answer. b

(200). Funicular hernia is a type of?

a. Direct inguinal hernia

b. Indirect inguinal hernia

c. Femoral hernia

d. Umblical hernia

Solution. (b) Indirect inguinal herniaRef: Read the text belowSol : Three types of indirect inguinal hernia occur- Bubonocele : The hernia is limited to the inguinal canal.- Funicular : The processus vaginalis is closed just above the epididymis. The contents of the sac can be felt separately from the testis,which lies below the hernia.- Complete : A complete inguinal hernia is rarely present at birth but is commonly encountered in infancy. It also occurs in adolescenceor in adulthood. The testis appears to lie within the lower part of the hernia.

Correct Answer. b

Copyright © 2014 Delhi Academy of Medical Sciences, All Rights Reserved. 86/86

Test Answer 1.(b) 2.(d) 3.(a) 4.(b) 5.(c) 6.(d) 7.(c) 8.(b) 9.(b) 10.(c)

11.(b) 12.(c) 13.(d) 14.(b) 15.(b) 16.(c) 17.(c) 18.(a) 19.(b) 20.(b)

21.(c) 22.(a) 23.(b) 24.(b) 25.(b) 26.(b) 27.(c) 28.(a) 29.(a) 30.(a)

31.(b) 32.(c) 33.(a) 34.(b) 35.(d) 36.(c) 37.(b) 38.(b) 39.(b) 40.(d)

41.(b) 42.(c) 43.(b) 44.(b) 45.(d) 46.(a) 47.(a) 48.(a) 49.(d) 50.(a)

51.(c) 52.(d) 53.(d) 54.(b) 55.(b) 56.(c) 57.(a) 58.(a) 59.(c) 60.(a)

61.(d) 62.(b) 63.(d) 64.(a) 65.(d) 66.(a) 67.(d) 68.(b) 69.(d) 70.(a)

71.(a) 72.(c) 73.(d) 74.(a) 75.(a) 76.(a) 77.(c) 78.(a) 79.(d) 80.(b)

81.(b) 82.(a) 83.(d) 84.(a) 85.(c) 86.(c) 87.(b) 88.(a) 89.(d) 90.(c)

91.(b) 92.(d) 93.(d) 94.(d) 95.(a) 96.(a) 97.(b) 98.(a) 99.(c) 100.(d)

101.(a) 102.(c) 103.(d) 104.(c) 105.(d) 106.(d) 107.(c) 108.(d) 109.(b) 110.(c)

111.(c) 112.(c) 113.(c) 114.(b) 115.(c) 116.(d) 117.(b) 118.(b) 119.(a) 120.(a)

121.(a) 122.(d) 123.(a) 124.(c) 125.(d) 126.(c) 127.(c) 128.(b) 129.(a) 130.(a)

131.(b) 132.(c) 133.(b) 134.(b) 135.(a) 136.(a) 137.(b) 138.(c) 139.(c) 140.(a)

141.(a) 142.(b) 143.(b) 144.(d) 145.(c) 146.(b) 147.(d) 148.(c) 149.(b) 150.(c)

151.(b) 152.(a) 153.(d) 154.(d) 155.(b) 156.(a) 157.(a) 158.(b) 159.(c) 160.(b)

161.(c) 162.(d) 163.(d) 164.(c) 165.(b) 166.(b) 167.(b) 168.(b) 169.(b) 170.(b)

171.(a) 172.(b) 173.(d) 174.(b) 175.(c) 176.(c) 177.(a) 178.(d) 179.(c) 180.(b)

181.(c) 182.(a) 183.(d) 184.(b) 185.(b) 186.(a) 187.(b) 188.(a) 189.(d) 190.(d)

191.(b) 192.(a) 193.(d) 194.(d) 195.(c) 196.(b) 197.(d) 198.(b) 199.(b) 200.(b)